download scanned papers here - Paper 1    Paper II


UPSC PRELIMS 2021 - PAPER I - ANSWER KEY

Exam Analysis HOME       2021 - Paper I - English      Paper II - E       Paper I - Hindi       Paper II - H      



BIRD'S EYEVIEW

Exam Analysis HOME       2021 - Paper I - English      Paper II - E       Paper I - Hindi       Paper II - H      

  • Basics: The UPSC Civil Services Prelims 2021 were conducted across 77 cities in the country on October 10, 2021. Nearly 10 lakh candidates had registered for the examination this year as well. About 5-5.5 lakh candidates were estimated to appear in the exams. The number of seats has been reduced this year. Last year, it was 796. This year it is 712.

  • 2021 was a tough paper, though not as tough as that in 2020 (toughest in previous seven years). A good rate of attempt is – Paper I – 65-70 questions; Paper II – 27+ questions

  • The Prelims 2021 was tough, but unlike that in 2020 which proved to be a big challenge to anyone – fresh or experienced – who either expected a set pattern based on the year 2019 or earlier years

  • By now, students know they have to be prepared for anything and everything

  • Lesson (once more): Long term tapasya, going in-depth, no keep hitting more, discretion, practice. Since it’s a relative test, ultimately those who kept their cool and did their best, will make it

  • Topic-wise breakup and flavour:
    1. Relatively similar in difficulty level as is expected from a Civil Services exam
    2. Slightly more difficult then 2020 paper; Balanced Paper
    3. Questions from all subjects like Indian History, Economics, Polity, Environment, Indian and World Geography, Science and Technology, International Relations, Current Affairs etc.
    4. Maximum questions from history which was an amalgamation of ancient, medieval and modern
    5. Economy – Total 15 questions. 2 or 3 were easy and solved by simple elimination methods. Difficulty level was lower than last year. Certain question from static also.
    6. History –UPSC is going deeper each year. Questions connecting many aspects and history connecting with geography has been asked. Weight of medieval history questions increased (9 questions). Difficulty Level - Tough
    7. Polity – Some straightforward, some factual, some conceptual, some related to current affairs. Direct questions from Polity NCERT (class 11 and 12. For example: Constitutional government, Constitutional status of India in 1950, Devi Lal, essential features that indicates India's federal character)
    8. Geography – Factual questions, applied questions, many questions direct, easier than last year
    9. Environment - Questions with a bent towards traditional knowledge. Basic understanding of phylum, symbiotic relationships etc. The trend is same like more focus on the big organizations like UNEP, UNFCCC etc.
    10. Current Affairs – Past-6-12 months questions, Covid-19 Pandemic, India-China ties, 3 questions from sports were asked for the first time in the last 20 years! (In our categorization and break-up table, many of these may be found under General Science, Biology, S&T or Environment & Ecology, as they originally come from those topics)
  • Cut-offs: Most are expecting the cut-offs to be slightly higher than the last year, even if the number of vacancies is reduced. Some rough estimate can be – General 98.46± 5; EWS 84 ± 5; OBC 96 ± 5; SC 83 ± 5; ST 80 ± 5.
TOPICWISE DISTRIBUTION

Exam Analysis HOME       2021 - Paper I - English      Paper II - E       Paper I - Hindi       Paper II - H      


DETAILED QUESTIONWISE SOLUTIONS (Set A)

Exam Analysis HOME       2021 - Paper I - English      Paper II - E       Paper I - Hindi       Paper II - H      


1. Consider the following statements:

  1. The Governor of the Reserve Bank of India (RBI) is appointed by the Central Government.
  2. Certain provisions in the Constitution of India give the Central Government the right to issue directions to the RBI in the public interest.
  3. The Governor of the RBI draws his power from the RBI Act.

Which of the above statements are correct?

  1. 1 and 2 only
  2. 2 and 3 only
  3. 1 and 3 only
  4. 1, 2 and 3

Sol. Ans.(c).

Statements 1 and 3 are correct but 2 is not. Remember that entire structure of RBI came into being in 1935, as per the RBI Act, 1934.

The governor of RBI is indeed appointed by the government of India. So statement 1 is correct.

The RBI is not a constitutional body, as it was established under the Reserve Bank of India Act, 1934. The independence of the RBI is not constitutional. Though originally privately owned, since nationalisation in 1949, the Reserve Bank is fully owned by the Government of India. The Indian Constitution under the Seventh Schedule (Article 246) lays down the respective functions and financial resources of the Government at the Union and State level and contains three lists as know. In List I - Union List (97 Items), it does mention the RBI ( Functions: Defense, Atomic Energy and Mineral Resources, Foreign Affairs, Diplomatic Relations, Railways, Airways, Posts and Telegraph, Public Debt of the Union, Currency and Coinage, RBI, Banking, Insurance, Stock Exchanges, etc.) but does not give details of how power is to be exercised.

But as the RBI Act, 1934, per Chapter 2, clause 7, "Management" - (1) The Central Government may from time to time give such directions to the Bank as it may, after consultation with the Governor of the Bank, consider necessary in the public interest.

So statement 2 is not correct.

The governor indeed derives his powers from the RBI Act, 1934.

In October 2018, the government invoked never-before-used powers under the RBI Act allowing it to issue directions to the central bank governor on matters of public interest, in a development that gives a new twist to the ongoing skirmish between RBI and the government. The Section 7 of the RBI Act empowers the government to consult and give instructions to the governor to act on certain issues that the government considers serious and in public interest. This Section had never been used in independent India till now. It was not used even when the country was close to default in the dark days of 1991, nor in the aftermath of the 2008 crisis.

Statement 3 is correct.


2. With reference to casual workers employed in India, consider the following statements :

  1. All casual workers are entitled for Employees Provident Fund coverage.
  2. All casual workers are entitled for regular working hours and overtime payment.
  3. The government can by a notification specify that an establishment or industry shall pay wages only through its bank account.

Which of the above statements are correct?

  1. 1 and 2 only
  2. 2 and 3 only
  3. 1 and 3 only
  4. 1, 2 and 3

Sol. Ans.(d). As per the rules, in EPF, employee whose 'pay' is more than Rs 15,000 a month at the time of joining, is not eligible and is called non-eligible employee. Employees drawing less than Rs 15,000 a month have to mandatorily become members of the EPF. Then, in 2020, the Supreme Court held that an employer cannot differentiate between a casual worker and permanent employees, as far as social security benefits are concerned.

So statement 1 is correct. So option (b) is wrong.

The reforms in labour laws now mandate overtime payment even for casual workers. So statement 2 is also right.

Statement 3 is right.


3.Which among the following steps is most likely to be taken at the time of an economic recession?

  1. Cut in tax rates accompanied by increase in interest rate
  2. Increase in expenditure on public projects
  3. Increase in tax rates accompanied
  4. Reduction of expenditure on public projects
-

Sol. Ans.(b). When an economic recession strikes, the government will want to push economic activity. A good way will be to start many public projects, by investing government funds immediately. Option (a) is wrong as rising interest rates will slow credit uptake. Option (c) is wrong as rising tax rates will dampen investment sentiment. Option (d) is wrong as that will slow down economic recovery.


4. Consider the following statements:

Other things remaining unchanged, market demand for a good might increase if

  1. price of its substitute increases
  2. price of its complement increases
  3. the good is an inferior good and the income of the consumers increases its price falls
  4. Its prices fall

Which of the above statements are correct?

  1. 1 and 4 only
  2. 2, 3 and 4
  3. 1, 3 and 4
  4. 1, 2 and 3

Sol. Ans.(a). A substitute product serves the same purpose as another product in the market. Getting more of one means lower demand of other, i.e. a negative correlation. Consumption of one product replaces the other. So demand for one good will rise if the substitute product becomes costlier.

A complementary good adds value to another, and are used together. Cereals and milk go together. If one becomes costlier, other may suffer.

An inferior good – by definition – is a good whose demand drops when people's incomes rise!

Clearly, 1 and 4 will lead to rise in demand of the good, but 2 and 3 will not.


5. With reference to 'Urban Cooperative Banks' in India, consider the following statements :

  1. They are supervised and regulated by local boards set up by the State Governments.
  2. They can issue equity shares and preference shares.
  3. They were brought under the purview of the Banking Regulation Act, 1949 through an Amendment in 1966.

Which of the statements given above is/are correct?

  1. 1 only
  2. 2 and 3 only
  3. 1 and 3 only
  4. 1, 2 and 3

Sol. Ans.(b). RBI website informs that for augmentation of capital funds, UCBs are permitted to raise equity share capital by way of (i) issue of equity shares to persons within their area of operation enrolled as members, in accordance with the provisions of their bye-laws, and (ii) issue of additional equity shares to the existing members. UCBs are also permitted to issue various instruments to augment their capital by issuing Preference Shares. So 2 is right, so options (a) and (c) are wrong.

So 3 is surely right.

Regarding regulating the UCBs, RBI informs that it ‘regulates and supervises the banking functions of UCBs under the provisions of Banking regulation Act, 1949. Within the Reserve Bank, a separate department, viz. Urban Banks Department, has been entrusted with these functions. Urban Banks Department functions in close coordination with other regulators viz., Registrar of Cooperative Societies and Central Registrar of Cooperative Societies.

So statement 1 is not correct.


6. Indian Government Bond Yields are influenced by which of the following?

  1. Actions of the United States Federal Reserve
  2. Actions of the Reserve Bank of India
  3. Inflation and short-term interest rates

Select the correct answer using the code given below.

  1. 1 and 2 only
  2. 2 only
  3. 3 only
  4. 1, 2 and 3

Sol. Ans.(d). Al three can affect Indian government bond yields.

Bond yield means the returns an investor will derive by investing in the bond. The mathematical formula for calculating yield is the annual coupon rate divided by the current market price of the bond.

The reason bond yields change (rise or fall) could be due to good or bad outcomes in the economy. It could be due to inflation from weaker rupee and hence more expensive oil imports, or consumption-led inflation where demand increase or supply shocks due to low production during the lockdown, or due to expected higher default risk and perhaps lower credit rating, or could be people selling risk-free bonds for risky equity. Usually it’s a mix of one of these as well as the overall supply-demand of government bonds (something that the RBI can control).

If the US Treasury yields continue to rise, inflation continues to climb, and an improving economy pushes central banks to change policy stance, then it may lead to massive foreign fund outflows, forcing India’s share market into further correction.


7. Consider the following:

  1. Foreign currency convertible bonds of
  2. Foreign institutional investment with certain conditions
  3. Global depository receipts
  4. Non-resident external deposits

Which of the above can be included in Foreign Direct Investments?

  1. 1, 2 and 3
  2. 3 only
  3. 2 and 4
  4. 1 and 4

Sol. Ans.(d). The foreign currency convertible bonds (FCCBs) are bonds invested in an Indian company. Since these bonds are convertible in to equity shares over a period of time as provided in the instrument, therefore they are covered under FDI policy. So 1 is part of FDI. Options (b) and (c) are ruled out.

Now, 3 is definitely not FDI, but FPI, hence option (a) is ruled out.

‘Non-resident entity’ means a ‘person resident outside India’ as defined under FEMA. ‘Non-Resident Indian’ (NRI) means an individual resident outside India who is a citizen of India. NRED are considered FDI.

Hence, (d).


8. Consider the following statements:

The effect of devaluation of a currency is that it necessarily

  1. Improves the competitiveness of the domestic exports in the foreign markets
  2. increases the foreign domestic currency value
  3. improves the trade balance of

Which of the above statements is/are correct?

  1. 1 only
  2. 1 and 2
  3. 3 only
  4. 2 and 3

Sol. Ans.(a). Devaluation means there is a fall in the value of a currency. The main effects are (i) exports are cheaper to foreign customers, (ii) imports more expensive, and (iii) in the short-term, a devaluation tends to cause inflation, higher growth and increased demand for exports. So 1 is correct. Hence options (c) and (d) are ruled out.

It actually decreases the foreign value of domestic currency, so 2 is not true. Only possible option is Option (a).


9. Which one of the following effects of the creation of black money in India has been the main cause of worry to the Government of India?

    1. Diversion of resources to the purchase of real estate and investment in luxury housing
    2. Investment in unproductive activities and purchase of precious stones, jewellery, gold, etc.
    3. Large donations to political parties and growth of regionalism
    4. Loss of revenue to the State Exchequer due to tax evasion

Sol. Ans.(d).

All four options sound enticing enough, but (a) and (b) do not affect the government as much as (d) surely does. If the government revenues fall, it is a direct hit. Option (c) is not a concern because all political parties enjoy large donations, if possible. The use of “regionalism” makes (c) wrong, too.


10. Which one of the following is likely to be the most inflationary in its effects?

  1. Repayment of public debt
  2. Borrowing from the public to finance a budget deficit
  3. Borrowing from the banks to finance a budget deficit
  4. Creation of new money to finance a budget deficit

Sol. Ans.(d).

In option (d), i.e., creation of new money to finance a budget deficit will increase the supply of currency notes, which is a reason for the inflation. When the supply of money increases in the market, the customer demand automatically increases, which cause the price rise.

Budget deficit happens when total expenditure exceeds total revenue. Deficit financing is a tool to generate funds in order to gap the budget deficit. This can be done in three ways, first by creating new money; second by borrowing from internal sources (ex. RBI, by issuing bonds etc.) and third by borrowing from external sources (ex. WB, IMF etc.).


11. The money multiplier in an economy increases with which one of the following?

  1. Increase in the Cash Reserve Ratio in the banks
  2. Increase in the Statutory Liquidity Ratio in the banks
  3. Increase in the banking habit of the people
  4. Increase in the population of the country

Sol. Ans.(d). The Money Multiplier refers to how an initial deposit can lead to a bigger final increase in the total money supply. For example, if the commercial banks gain deposits of Rs.1 crore and this leads to a final money supply of Rs.100 crore, the money multiplier is 10.

If more people start banking, then credit will rise and money multiplier too will.

As for the CRR, the smaller the cash reserve ratio is, the larger the increase it brings to the money supply, because more of the customers' deposits get loaned out by the bank. So higher CRR means lower money multiplier. Same holds for SLR also.


12. With reference to Indian economy, demand-pull inflation can be cluby increased by which of the following?

  1. Expansionary policies
  2. Fiscal stimulus
  3. Inflation-indexing wages
  4. Higher purchasing power
  5. Rising interest rates

Select the correct answer using the code given below.

  1. 1, 2 and 4 only
  2. 3, 4 and 5 only
  3. 1, 2, 3 and 5 only
  4. 1, 2, 3, 4 and 5

Sol. Ans.(a). Fiscal stimulus puts money in people’s hands. It is bound to lead to demand-pull inflation. So option (b) is wrong. Similarly 4 is surely correct, so option (c) is wrong.

Focus on 5 – rising interest rates. That will not lead to more demand (via more credit) but less demand.

Hence, (d) is gone. Hence (a) is right!

Short Cut: If you can identify that 5 is wrong, then three options (b, c, d) get ruled out instantly.


13.With reference to India, consider the following statements :

  1. Retail investors through demat account can invest in 'Treasury Bills' and 'Government of India Debt Bonds' in primary market.
  2. The 'Negotiated Dealing System-Order Matching' is a government securities trading platform of the Reserve Bank of India.
  3. The 'Central Depository Services Ltd.' is jointly promoted by the Reserve Bank of India and the Bombay Stock Exchange.

Which of the statements given above is/are correct?

  1. 1 only
  2. 1 and 2
  3. 3 only
  4. 2 and 3

Sol. Ans.(b).

Both 1 and 2 are correct. Retail investors are allowed to invest in primary market of T-bills and G-secs.

The Negotiated Dealing System, or NDS, is an electronic trading platform operated by the Reserve Bank of India to allow issuing and exchange of government securities and other money market instruments. The goal was to cut manual paperwork, and raise transparency for all participants.

CDSL was initially promoted by Bombay Stock Exchange Ltd (BSE Ltd) - the Asia's latest Stock Exchange. Other shareholders include HDFC Bank Ltd, Standard Chartered Bank and Canara Bank. The RBI was not involved. So, 3 is wrong. So (c) and (d) are ruled out.


14.With reference to 'Water Credit', consider the following statements :

  1. It puts micro finance tools to work in the water and sanitation sector.
  2. It is a global initiative launched under the aegis of the World Health Organization and the World Bank.
  3. It aims to enable the poor people to meet their water needs without depending on subsidies.

Which of the statements given above are correct?

  1. 1 and 2 only
  2. 2 and 3 only
  3. 1 and 3 only
  4. 1, 2 and 3

Sol. Ans.(c).

Water.org is a global nonprofit organization working to bring water and sanitation to the world. It wants to make it safe, accessible, and cost-effective. It is not associated with WHO or WB. (Hence three options can be ruled out directly – a, b and d)

It helps people get access to safe water and sanitation through affordable financing, such as small loans.

A key barrier to safe water and sanitation is affordable financing. The WaterCredit Initiative® loan program addresses this barrier head-on. WaterCredit helps bring small loans to those who need access to affordable financing and expert resources to make household water and toilet solutions a reality.

WaterCredit is a powerful solution and the first to put microfinance tools to work in the water and sanitation sector. WaterCredit helps bring small loans to those who need access to affordable financing and expert resources to make household water and toilet solutions a reality.

Short Cut: If you can identify that 2 is wrong, then three options (a, b, d) get ruled out instantly.


15.In India, the central bank's function as the 'lender of last resort' usually refers to which of the following?

  1. Lending to trade and industry bodies when they fail to borrow from other sources
  2. Providing liquidity to the banks having a temporary crisis
  3. Lending to governments to finance budgetary deficits

Select the correct answer using the code given below.

  1. 1 and 2
  2. 2 only
  3. 2 and 3
  4. 3 only

Sol. Ans.(b). The RBI clearly mentions “As a Banker to Banks, the Reserve Bank also acts as the ‘lender of the last resort’. It can come to the rescue of a bank that is solvent but faces temporary liquidity problems by supplying it with much needed liquidity when no one else is willing to extend credit to that bank. The Reserve Bank extends this facility to protect the interest of the depositors of the bank and to prevent possible failure of the bank, which in turn may also affect other banks and institutions and can have an adverse impact on financial stability and thus on the economy.”

RBI does not act as lender of last resort for deficit monetization of union government.



16.'R2 Code of Practices' constitutes a tool available for promoting the adoption of

  1. environmentally responsible practices in electronics recycling industry
  2. ecological management of `Wetlands of International Importance' under the Ramsar Convention
  3. sustainable practices in the cultivation of agricultural crops in degraded lands
  4. 'Environmental Impact Assessment' in the exploitation of natural resources

Sol. Ans.(a). R2 stands for Responsible Recycling and is a standard specifically created for the electronics recycling industry by Sustainable Electronics Recycling International (SERI). The standards were then accredited by ANAB, and in 2008, R2 was released. {The ANSI National Accreditation Board (ANAB) is a non-governmental organization that provides accreditation services and training to public- and private-sector organizations, serving the global marketplace. ANAB is the largest accreditation body in North America and provides services in more than 75 countries.}


17.Why is there a concern about copper smelting plants?

  1. They may quantities of release lethal carbon monoxide into environment.
  2. The copper slag can cause the leaching of some heavy metals into environment.
  3. They may release sulphur dioxide as a pollutant.

Select the correct answer using the code given below.

  1. 1 and 2 only
  2. 2 and 3 only
  3. 1 and 3 only
  4. 1, 2 and 3

Ans.(b). Statement 1 is not a major concern, hence three options are ruled out.

The basic copper smelting reaction Cu2O + CO → 2Cu + CO2 produces copper and volatile carbon dioxide and nothing else.

Smelting means to rip apart some molecule that contains a metal. It will be broken up into the elemental metal and something else. We “reduce” the molecule in a reduction process (mostly by reacting it with some reducing agent like carbon monoxide (CO)).

Mined ores are processed to concentrate the minerals of interest. In the case of metal ores, these mineral concentrates usually need to be further processed to separate the metal from other elements in the ore minerals. Smelting is the process of separating the metal from impurities by heating the concentrate to a high temperature to cause the metal to melt (along with a solid waste product called slag)

The main sources of pollution caused by smelting are contaminant-laden air emissions and process wastes such as wastewater and slag. It releases toxic metals like copper, silver, iron, cobalt and selenium into the atmosphere. Smelters also release gaseous sulfur dioxide, contributing to acid rain, which acidifies soil and water.

The smelting of sulfide ores results in the emission of sulfur dioxide gas, which reacts chemically in the atmosphere to form a sulfuric acid mist. As this acid rain falls to the earth, it increases the acidity of soils, streams, and lakes, harming the health of vegetation and fish and wildlife populations

The smelting process accounts for about 17 percent of greenhouse gas emissions from the aluminum industry and 1 percent of total global CO2 emissions.

Short Cut: If you can identify that 1 is wrong, then three options (a, c, d) get ruled out instantly.


18. With reference to furnace oil, consider the following statements :

  1. It is a product of oil refineries.
  2. Some industries use it to generate power.
  3. Its use causes sulphur emissions into environment.

Which of the statements given above are correct?

  1. 1 and 2 only
  2. 2 and 3 only
  3. 1 and 3 only
  4. 1, 2 and 3

Ans.(d). Home heating oil or furnace oil is a by-product of crude oil got from the distillation of petroleum. It can be confused with diesel due to overlapping properties. But they are different.

It consists mainly of residues from crude-oil distillation. It is used primarily for steam boilers in power plants, aboard ships, and in industrial plants. Commercial fuel oils usually are blended with other petroleum fractions to produce the desired viscosity and flash point.

Residual fuel oil (furnace oil) is less useful because it is so viscous that it has to be heated with a special heating system before use and it may contain relatively high amounts of pollutants, particularly sulfur, which forms sulfur dioxide upon combustion.


19. What is blue carbon?

  1. Carbon captured by oceans and coastal ecosystems
  2. Carbon sequestered in forest biomass and agricultural soils
  3. Carbon contained in petroleum and natural gas
  4. Carbon present in atmosphere

Ans.(a). Blue carbon is carbon captured by the world's ocean and coastal ecosystems. Human activities emit carbon dioxide, that contains atmospheric carbon. Gases are changing the world's climate, and not in a good way. But the ocean and coasts provide a natural way of reducing the impact of greenhouse gases on our atmosphere, through sequestration (or taking in) of this carbon.

Sea grasses, mangroves, and salt marshes along our coast "capture and hold" carbon, acting as a carbon sink. These coastal systems, though much smaller in size than the planet's forests, sequester this carbon at a much faster rate, and can continue to do so for millions of years. Most of the carbon taken up by these ecosystems is stored below ground. The carbon found in coastal soil is often thousands of years old!


20. In the nature, which of the following is/are most likely to be found surviving on a surface without soil?

  1. Fern
  2. Lichen
  3. Moss
  4. Mushroom

Select the correct answer using the code given below.

  1. 1 and 4 only
  2. 2 only
  3. 2 and 3
  4. 1, 3 and 4

Ans.(c). Lichens are a complex life form that is a symbiotic partnership of two separate organisms, a fungus and an alga. The dominant partner is the fungus.

Lichens grow on any undisturbed surface--bark, wood, mosses, rock, soil, peat, glass, metal, plastic, and even cloth. Lichens can absorb water through any part of their thalli and have no need of roots. Lichens do not damage plants or rob the bark of moisture.

So 2 is right, and options (a) and (d) are ruled out.

Mosses are non-flowering plants which produce spores and have stems and leaves, but don't have true roots. Mosses, and their cousins liverworts and hornworts, are classified as Bryophyta (bryophytes) in the plant kingdom.

Mosses may grow on damp soil, tree bark, rocks, concrete, or almost any other reasonably stable surface.

Ferns require indirect sunlight, moist soil, and a humid atmosphere. So (a) and (d) are ruled out. Hence (c) is chosen.


21.Which one of the following is used in preparing a natural mosquito repellent?

  1. Congress grass
  2. Elephant grass
  3. Lemongrass
  4. Nut grass

Ans.(c). It is the Lemongrass.

Parthenium hysterophorus, known as “Congress grass” in India, has spread to more than 40 tropical and subtropical countries, earning its place in the list of the world’s 100 worst invasive species. The alien has invaded almost all Indian states as well, as per estimates, its invasion has resulted in yield losses of up to 40% in several crops in the country and has also caused a 90% drop in forage production.

Elephant grass (Pennisetum purpureum Schumach.) is a major tropical grass. It is one of the highest yielding tropical grasses. It is a very versatile species that can be grown under a wide range of conditions and systems: dry or wet conditions, smallholder or larger scale agriculture.

Lemongrass (Cymbopogon) is a plant, the leaves and the oil of which are used to make medicine. It is used for treating digestive tract spasms, stomachache, high blood pressure, convulsions, pain, vomiting, cough, achy joints (rheumatism), fever, the common cold, and exhaustion. It is also used to kill germs and as a mild astringent. The Lemongrass plant is known to be an excellent mosquito repellant.

Nut grass or Nagarmotha (Cyperus rotundus) is used in the treatment and prevention of many ailments. Rhizomes of the plant are used to treat diarrhea, intestinal parasites, indigestion and bowel disorders.


22.Consider the following kinds of organisms :

  1. Copepods
  2. Cyanobacteria
  3. Diatoms
  4. Foraminifera

Which of the above are primary producers in the food chains of oceans?

  1. 1 and 2
  2. 2 and 3
  3. 3 and 4
  4. 1 and 4

Ans.(b). In oceans, the primary producers — including bacteria, phytoplankton, and algae — form the lowest trophic level, the base of the aquatic food web. They synthesize their own energy without needing to eat. Many photosynthesize, using the sun's energy to build carbohydrates.

Copepods are major secondary producers in the world ocean. In aquatic environments, cyanobacteria are important primary producers, as said. Diatoms are one of the major primary producers in the ocean, and are photosynthesising algae.

So 2 and 3 are right (cyanobacteria, diatoms). Hence, (a) is wrong, (c) is wrong and (d) is wrong.

Short cut: Once you know either of 2 or 3, or both, things become easy.


23.Consider the following animals :

  1. Hedgehog
  2. Marmot
  3. Pangolin

To reduce the chance of being captured by predators, which of the above organisms rolls up/roll up and protects/ protect its/their vulnerable parts?

  1. 1 and 2
  2. 2 only
  3. 3 only
  4. 1 and 3

Ans.(d). A hedgehog is a spiny mammal. To protect itself, a hedgehog curls his head and feet into his body and forms a ball to protect his soft, delicate underside. So 1 is right. So (b) and (c) are wrong.

Marmot (genus Marmota) are any of 14 species of giant ground squirrels found primarily in North America and Eurasia. These rodents are large and heavy. Upon seeing a predator, the yellow-bellied marmot whistles to warn the others in the area, after which it typically hides in a nearby rock pile until there is no more threat.

If under threat, a pangolin will immediately curl into a tight ball and will use their sharp-scaled tails to defend themselves. So (d) is right.


24.With reference to the 'New York Declaration on Forests', which of the following statements are correct?

  1. It was first endorsed at the United Nations Climate Summit in 2014.
  2. It endorses a global timeline to end the loss of forests.
  3. It is a legally binding international declaration.
  4. It is endorsed by governments, big companies and indigenous communities. /li>
  5. India was one of the signatories at its inception.

Select the correct answer using the code given below.

  1. 1, 2 and 4
  2. 1, 3 and 5
  3. 3 and 4
  4. 2 and 5

Ans.(a). Since it is voluntary and not compulsory, options (b) and (c) are rejected.

The New York Declaration on Forests was endorsed at the 2014 Climate Summit by more than 150 governments, companies, indigenous peoples and civil society organizations committed to doing their part to achieve the Declaration’s ten goals and follow its accompanying action agenda.

Its ten goals include halting natural forest loss by 2030, restoring 350 million hectares of degraded landscapes and forestlands, improving governance, increasing forest finance, and reducing emissions from deforestation and forest degradation as part of a post-2020 global climate agreement.

India is not a signatory. So 5 is wrong, so (b) and (d) are wrong.

Short cut: Knowing all 5 things is not necessary. If you knew any two, it could be solved.


25.Magnetite particles, suspected to cause neurodegenerative problems, are generated as environmental pollutants from which of the following?

  1. Brakes of motor vehicles
  2. Engines of motor vehicles
  3. Microwave stoves within homes
  4. Power plants /li>
  5. Telephone lines Select the correct answer using the code given below.
  1. 1,2,3 and 5 only
  2. 1,2 and 4 only
  3. 3,4 and 5 only
  4. 1,2,3,4 and 5

Ans.(b). Magnetite is a mineral and one of the main iron ores, with the chemical formula Fe3O4.

Magnetite nanospheres, formed by combustion and/or friction-derived heating, are prolific in urban, airborne particulate matter (PM). As many of the airborne magnetite pollution particles are <200 nm in diameter, they can enter the brain directly through the olfactory nerve and by crossing the damaged olfactory unit. This discovery is important because nanoscale magnetite can respond to external magnetic fields, and is toxic to the brain.

Vehicle brake systems are the major source of airborne magnetite at the roadside. Magnetite concentration is similar in diesel- and petrol-engine exhaust PM. High magnetite content of vehicle brake wear might constitute a risk to human health.

So 1 and 2 are correct, and option (c) is wrong.

There is no reported incidence of magnetite generated from home microwave stoves. So 3 is wrong. So options (a), (c) and (d) are wrong.


26.Which one of the following is a filter

  1. Catfish
  2. Octopus
  3. Oyster
  4. Pelican

Ans.(c). Oysters are filter feeders. They eat by pumping large volumes of water through their body. Water is pumped through the oyster’s gills by the beating of cilia. Plankton, algae and other particles become trapped in the mucus of the gills. From there these particles are transported to the oyster’s mouth and esophagus to be eaten, then to the stomach to be digested.

Catfish and Octopus can be ruled out directly, as both are regular eaters. Most catfish are actually bottom feeders.

Pelicans mainly eat fish, but they are opportunistic feeders and eat a variety of aquatic animals including crustaceans, tadpoles and turtles. When food is caught, the pelican manipulates it in its bill until the prey typically has its head pointing down the pelican's throat.


27.In case of which one of the following biogeochemical cycles, the weathering of rocks is the main source of release of nutrient to enter the cycle?

  1. Carbon cycle
  2. Nitrogen cycle
  3. Phosphorus cycle
  4. Sulphur cycle

Ans.(c) or (d). A Biogeochemical cycle is a natural pathway by which essential elements of living matter are circulated.

The carbon cycle describes the process in which carbon atoms continually travel from the atmosphere to the Earth and then back into the atmosphere. Carbon is released back into the atmosphere when organisms die, volcanoes erupt, fires blaze, fossil fuels are burned, and through a variety of other mechanisms. So (a) is wrong.

Nitrogen Cycle is a biogeochemical process through which nitrogen is converted into many forms, consecutively passing from the atmosphere to the soil to organism and back into the atmosphere. There are five stages in the nitrogen cycle: fixation or volatilization, mineralization, nitrification, immobilization, and denitrification. So (b) is wrong.

The phosphorus cycle is the process by which phosphorus moves through the lithosphere, hydrosphere, and biosphere. Phosphorus is essential for plant and animal growth, as well as the health of microbes inhabiting the soil, but is gradually depleted from the soil over time. Much of the phosphorus on Earth is tied up in rock and sedimentary deposits, from which it is released by weathering, leaching, and mining.

The sulfur (sulphur) cycle describes the movement of sulfur through the geosphere and biosphere. Sulfur is released from rocks through weathering, and then assimilated by microbes and plants. It is then passed up the food chain and assimilated by plants and animals, and released when they decompose.


28.Which of the following are detritivores?

  1. Earthworms
  2. Jellyfish
  3. Millipedes
  4. Seahorses
  5. Woodlice Select the correct answer using the code given below.
  1. 1, 2 and 4 only
  2. 2, 3, 4 and 5 only
  3. 1, 3 and 5 only
  4. 1, 2, 3, 4 and 5

Ans.(c). A detritivore is an organism that eats dead or decaying plants or animals as food. It is a heterotrophic organism, which obtains its nutrition by feeding on detritus.

Detritivores are often invertebrate insects such as mites, beetles, butterflies and flies; mollusks such as slugs and snails; or soil-dwelling earthworms, millipedes and woodlice.

Fish, jellyfish (2) and crustaceans are common secondary consumers. Jellyfish feed on phytoplankton and zooplanktons.

Shortcut: Three options have 2 in them, so are rejected.

29.The 'Common Carbon Metric', supported feeder? by UNEP, has been developed for

  1. assessing the carbon footprint of building operations around the world
  2. enabling commercial farming entities around the world to enter carbon emission trading
  3. enabling governments to assess the overall carbon footprint caused by their countries
  4. assessing the overall carbon foot-print caused by the use of fossil fuels by the world in a unit time

Ans.(a). The Common Carbon Metric is the calculation used to define measurement, reporting, and verification for GHG emissions associated with the operation of buildings types of particular climate regions. It does not include value-based interpretation of the measurements such as weightings or benchmarking.

UNEP said that the purpose of a Common Carbon Metric for buildings was to give the sector that represents 40% of the world’s energy consumption and related 1/3rd of global greenhouse gas (GHG) emissions a tool that doesn’t exist today – a way to measure, report, and verify reductions in a consistent and comparable way


30.Which of the following have species that can establish symbiotic relationship with other organisms?

  1. Cnidarians
  2. Fungi
  3. Protozoa

Select the correct answer using the code given below.

  1. 1 and 2 only
  2. 2 and 3 only
  3. 1 and 3 only
  4. 1, 2 and 3

Ans.(d). Symbiosis Symbiosis is a close ecological relationship between the individuals of two (or more) different species.

The relationship between cnidarians and dinoflagellate algae is termed as "symbiotic".

(Cnidaria is a phylum under kingdom Animalia containing over 11,000 species of aquatic animals found both in freshwater and marine environments)

Two common mutualistic relationships involving fungi are mycorrhiza and lichen.

Symbiosis in protozoa mostly represents a close mutualistic association between a protozoan and unicellular symbionts (bacteria, cyanobacteria or/and unicellular algae) or protozoans and a multicellular organism (ruminants, lower termites, wood-eating cockroaches, plants)


31.With reference to Chausath Yogini Temple situated near Morena, consider the following statements:

  1. It is a circular temple built during the reign of Kachchhapaghata Dynasty.
  2. It is the only circular temple built in India.
  3. It was meant to promote the Vaishnava cult in the region.
  4. Its design has given rise to a popular belief that it was the inspiration behind the Indian Parliament building.

Which of the statements given above are correct?

  1. 1 and 2
  2. 2 and 3 only
  3. 1 and 4
  4. 2,3 and 4.

Ans.(c). The Chausath Yogini Temple, Mitavli, is one of the rarest of Hindu temples in India. Its plan is circular and it is dedicated to the Tantric sect of Chausath Yogini. Most of the Hindu temples are based on square or rectangular plan.

India’s Parliament house was inspired by the circular Chausath Yogini Temple.

So, 4 is correct. So, options (a) and (b) are wrong. Since this is not the only circular temple, hence 2 is wrong. Hence (c) is right.

According to an inscription dated to 1323 CE (Vikram Samvat 1383), the temple was built by the Kachchhapaghata king Devapala (r. c. 1055 – 1075). It is said that the temple was the venue of providing education in astrology and mathematics based on the transit of the Sun. It is externally circular in shape with a radius of 170 feet and within its interior part it has 64 small chambers. Within the main central shrine there are slab coverings which have perforations in them to drain rainwater to a large underground storage. The pipe lines from the roof lead the rain water to the storage are also visible.

The cells and the main shrine are flat topped, but it is believed that initially each had a shikhara on top. While the 64 Yoginis originally placed in the 64 subsidiary shrines are now missing, a Shiva linga has taken their places in each cell. The central shrine also holds a Shivalinga.


32.Which one of the following ancient towns is well-known for its elaborate system of water harvesting and management by building a - series of dams and channelizing water into connected reservoirs?

  1. Dholavira
  2. Kalibangan
  3. Rakhigarhi
  4. Ropar

Ans.(a). This was a very simple and direct question.

The configuration of the city of Dholavira, during its heyday, is an example of planned city with planned and segregated urban residential areas based on possibly differential occupational activities, and a stratified society. Technological advancements in water harnessing systems, water drainage systems as well architecturally and technologically developed features are reflected in the design, execution, and effective harnessing of local materials.

Unlike other Harappan antecedent towns normally located near to rivers and perennial sources of water, the location of Dholavira in the island of Khadir was strategic to harness different mineral and raw material sources (copper, shell, agate-carnelian, steatite, lead, banded limestone, among others) and to facilitate internal as well as external trade to the Magan (modern Oman peninsula) and Mesopotamian regions.


33.In the first quarter of seventeenth Temple situated near Morena, consider century, in which of the following the following statements : was/were the factory/factories of the English East India Company located?

  1. Broach
  2. Chicacole
  3. Trichinopoly

Select the correct answer using the code given below.

  1. 1 only
  2. 1 and 2
  3. 3 only
  4. 2 and 3

Ans.(a). By 1623, the English East India Company had established factories at Surat, Broach, Ahmedabad, Agra, and Masulipatam. In 1625, the East India Company's authorities at Surat made an attempt to fortify their factory, but the chiefs of the English factory were immediately imprisoned and put in irons by the local authorities of the Mughal Empire.

The English opened their first factory in the South at Masulipatam in 1611. But they soon shifted the center of their activity to Madras the lease of which was granted to them by the local king in 1639.

>

34.From the decline of Guptas until the rise of Harshavardhana in the early seventh century, which of the following kingdoms were holding power in Northern India?

  1. The Guptas of Magadha
  2. The Paramaras of Malwa
  3. The Pushyabhutis of Thanesar
  4. The Maukharis of Kanauj
  5. The Yadavas of Devagiri
  6. The Maitrakas of Valabhi

Select the correct answer using the code given below.

  1. 1, 2 and 5
  2. 1, 3, 4 and 6
  3. 2, 3 and 4
  4. 5 and 6

Ans.(b). After the decline of the Gupta Empire, northern India had four empires – (i) Guptas of Magadha, (ii) the Pushyabhutis, (iii) the Maukharis, and (iv) the Maitrakas.

So 1 is correct and 3 is correct. So (a), (c) and (d) are wrong. So (b) is our answer.

Also note that the Yadavas of Deogiri belonged to the Medieval Era. So 5 is wrong. So options (a) and (d) are wrong.


35.According to Portuguese writer Nuniz, the women in Vijayanagara Empire were expert in which of the following areas?

  1. Wrestling
  2. Astrology
  3. Accounting
  4. Soothsaying

Select the correct answer using the code given below.

  1. 1, 2 and 3 only
  2. 1, 3 and 4 only
  3. 2 and 4 only
  4. 1, 2, 3 and 4

Ans.(d). The Vijayanagara Empire was founded by Harihara and Bukka and the reign was from 1336 A.D to 1646 A.D.

Women occupied a high position and took an active part in the political, social and literary life of the empire. They were educated and trained in wrestling, in the use of various weapons of offence and defence, in music and fine arts. Some women also received an education of high order. Nuniz writes that the kings had women astrologers, clerks, accountants, guards and wrestlers.

From https://www.thehindubusinessline.com/news/variety/Alpha-women-of-Hampi/article20396779.ece

Women occupied a respected position in Vijayanagar society. Besides accomplished musicians and dancers, women excelled as astrologers, soothsayers, judges, writers and eminent litterateurs, and many were actively involved in matters of administration and trade.


36.With reference to Madanapalle of Andhra Pradesh, which one of the following statements is correct?

  1. Pingali Venkayya designed the tricolour Indian National Flag here.
  2. Pattabhi Sitaramaiah led the Quit India Movement of Andhra region from here.
  3. Rabindranath Tagore translated the National Anthem from Bengali to English here.
  4. Madame Blavatsky and Colonel Olcott set up headquarters of Theosophical Society first here.

Ans.(c). Rabindranath Tagore translated "Jana Gana Mana" from Bengali to English and also set it to music in Madanapalle.

The National Anthem was written as early as in 1911 and was sung at the annual session of the Indian National Congress at Calcutta on 27 December that year. It was actually in Besant Theosophical College, Madanapalle, where Tagore stayed for a few days in February 1919 that the now familiar tune was set. It was Margaret Cousins, wife of educationist Dr. James Henry Cousins, who composed the tune for "Jana Gana Mana". Dr. J H Cousins was then the Principal of the Madanapalle College that was established by Dr. Annie Besant.


37.Consider the following pairs :


Which of the pairs given above is/are correctly matched?

  1. 1 only
  2. 1 and 2
  3. 3 only
  4. 2 and 3

Ans.(d). The Ganeshwar-Jodhpura Chalcolithic culture was first discovered in the 1970s after excavations at Jodhpura and Ganeshwar respectively. The site of Ganeshwar yielded a lot of copper objects such as arrowheads, spearheads, chisels, fish hooks, razor blades; ornaments like rings, hairpins, bangles, antimony rods, double spiral headed pin amongst others. So 3 is correct. So, (a) and (b) are wrong.

Around 50 km north-east of Kolkata, near the tiny village of Berachampa in West Bengal, lies the 2,300-year-old site of Chandraketugarh, filled with the most amazing terracotta sculptures.

Archaeologist Rakhaldas Banerji, who discovered the ruins of Mohenjodaro, visited Chandraketugarh and found a vast crop of terracotta artefacts. So 2 is also right, and (d) is our answer.

This question can be solved even if you don’t know about 1.

38. Consider the following statements :

  1. It was during the reign of Iltutmish that Chengiz Khan reached the Indus in pursuit of the fugitive Khwarezm prince.
  2. It was during the reign of Muhammad bin Tughluq that Taimur occupied Multan and crossed the Indus.
  3. It was during the reign of Deva Raya II of Vijayanagara Empire that Vasco da Gama reached the coast of Kerala.

Which of the statements given above is/are correct?

  1. 1 only
  2. 1 and 2
  3. 3 only
  4. 2 and 3

Ans.(a). In 1221, the great Mongol conqueror Genghis Khan stood victorious at western bank of the Indus river with nearly 50,000 soldiers, having just defeated Jalal-ad-Din, the Shah of Khwarazm. Jalal-ad-Din’s army was destroyed in this battle, which took place near the town of Kalabagh in Punjab. Northern India was ruled at the time by Shams ud-Din Iltutmish. So 1 is indeed right. So (c) and (d) are wrong.

The Battle of Delhi: In 1398 the Mongol-Turkish warrior Timur, ruler of Central Asia from his capital at Samarkand, found a reason to strike south, into India. A devout Muslim, Timur alleged that his co-religionist Sultan Nasiruddin Mahmud (not Tugluq) of Delhi was being too lenient toward his Hindu subjects. So, 2 is wrong. So answer is (a).


39.Consider the following statements :

  1. St. Francis Xavier was one of the founding members of the Jesuit Order.
  2. St. Francis Xavier died in Goa and a church is dedicated to him there.
  3. The Feast of St. Francis Xavier is celebrated in Goa each year.

Which of the statements given above are correct?

  1. 1 and 2 only
  2. 2 and 3 only
  3. 1 and 3 only
  4. 1, 2 and 3

Ans.(c). Born in Kingdom of Navarre (in Spain), he led a mission into Asia, mainly in the Portuguese Empire of the time and was influential in evangelization work, mostly in India.

Francis Xavier came to Goa, the centre of Portuguese activity in the East, on May 6, 1542. Much of the next three years he spent on the southeastern coast of India among the simple, poor pearl fishers, the Paravas. Francis travelled tirelessly from village to village instructing and confirming them in their faith. His evident goodness and the force of his conviction overcame difficulties of verbal communication.

He had come to realize that the way to the conversion of Japan lay through China; it was to the Chinese that the Japanese looked for wisdom. He never reached China, however. On December 3, 1552, Francis died of fever on the island of Sancian (Shangchuan, off the Chinese coast) as he attempted to secure entrance to the country, then closed to foreigners. So 2 is wrong, hence options (a), (b) and (d) are wrong.

Shortcut: If you know that 2 is wrong, the answer is directly found in (c).

40.With reference to the history of ancient India, which of the following statements is/are correct?

  1. Mitakshara was the civil law for upper castes and Dayabhaga was the civil law for lower castes.
  2. In the Mitakshara system, the sons can claim right to the property during the lifetime of the father, whereas in the Dayabhaga system, it is only after the death of the father that the sons can claim right to the property.
  3. The Mitakshara system deals with the matters related to the property held by male members only of a family, whereas the Dayabhaga system deals with the matters related to the property held by female members of a family.

Select the correct answer using the code given below:

  1. 1 and 2
  2. 2 only
  3. 1 and 3
  4. 3 only

Ans.(b). The Dayabhaga and the Mitakshara are two schools of law that govern the law of succession of the Hindu Undivided Family Under Indian Law.

Mitakshara law school – A joint family refers only to the male member of a family and extends to include his son, grandson and great-grandson. They collectively have co-ownership/Coparcenary in the Joint Family. Thus, a son by birth acquires an interest in the ancestral property of the joint family.

Dayabhaga law school – The son has no automatic ownership right by birth but acquires it on the demise of his father. Hence, 2 is surely correct. Hence, options (c) and (d) are wrong.

In the Mitakshara system the wife cannot demand partition. She, however, has the right to a share in any partition effected between her husband and her sons. Under the Dayabhaga this right does not exist for the women because the sons cannot demand partition as the father is the absolute owner. In both the systems, in any partition among the sons, the mother is entitled to a share equal to that of a son. Similarly, when a son dies before partition leaving the mother as his heir, the mother is entitled to a share of her deceased son as well as share in her own right when there is a partition between the remaining sons.


41.With reference to the history of ancient India, Bhavabhuti, Hast'malla and Kshemeshvara were famous

  1. Jain monks
  2. playwrights
  3. temple architects
  4. philosophers

Ans.(b). Bhavabhuti was an 8th-century scholar of India noted for his plays and poetry, written in Sanskrit.

His plays are considered the equal of the works of Kalidasa. Bhavabhuti seems to have been the court poet of King Yashovarman of Kanauj and known to have been in the entourage of the king who had fought with Lalitaditya Muktapada, the king of Kashmir and had got defeated by him

Hastimalla was a 13th century Kannada poets and writers in Hoysala Empire. He wrote the Purvapurana.

Kshemeshvara (990 – c. 1070 CE) was an 11th-century Sanskrit poet from Kashmir in India.

Shortcut: If you knew the profession of any one of these three, the question is solved.


42.Consider the following statements :

  1. The Montagu-Chelmsford Reforms of 1919 recommended granting voting rights to all the women above the age of 21
  2. The Government of India Act of 1935 gave women reserved seats in legislature.

Which of the statements given above is/are correct?

  1. 1 only
  2. 2 only
  3. Both 1 and 2
  4. Neither 1 nor 2

Ans.(b). In 1918, when Britain granted limited suffrage to women property holders, the law did not apply to British citizens in other parts of the Empire. Despite petitions presented by women, women's demands were ignored in the Montagu–Chelmsford Reforms. Though they were not granted voting rights, nor the right to stand in elections, the Government of India Act 1919 allowed Provincial Councils to determine if women could vote, provided they met stringent property, income, or educational levels. So 1 is wrong. So options (b) and (c) are wrong.

Between 1919 and 1929, all of the British Provinces, as well as most of the Princely states granted women the right to vote and, in some cases, allowed them to stand in local elections.

The report from the three Round Tables was sent to the Joint Committee of the British Parliament recommending lowering the voting age to 21, but retaining property and literacy restrictions, as well as basing women's eligibility on their marital status. It also provided special quotas for women and ethnic groups in provincial legislatures. These provisions were incorporated into the Government of India Act 1935. Though it extended electoral eligibility, the Act still allowed only 2.5% of the women in India to vote. So 2 is right. So option (b) is our answer.


43.With reference to 8th August, 1942 in Indian history, which one of the following statements is correct?

  1. The Quit India Resolution was adopted' by the AICC.
  2. The Viceroy's Executive Council was expanded to include more Indians.
  3. The Congress ministries resigned in seven provinces.
  4. Cripps proposed an Indian Union with full Dominion Status once the Second World War was over.

Ans.(a). On 8th August 1942, Mahatma Gandhi launched the Quit India Movement for freedom from British rule in Mumbai (then Bombay). The Quit India Movement, also known as the August Movement was a Civil Disobedience Movement launched by Gandhi for Satyagraha (independence).

On August 8, 1942, the Quit India Resolution was passed at the Bombay session of the All India Congress Committee (AICC). In his Quit India speech, that day at Gowalia Tank, Bombay, and Gandhi told Indians to follow non-violent civil disobedience. He told the masses to act as an independent nation.


44.Who among the following is associated with 'Songs from Prison', a translation of ancient Indian religious lyrics in English?

  1. Bal Gangadhar Tilak
  2. Jawaharlal Nehru
  3. Mohandas Karamchand Gandhi
  4. Sarojini Naidu

Ans.(a). It was Mahatma Gandhi who wrote this.


45.With reference to medieval India, which one of the following is the correct sequence in ascending order in terms of size?

  1. Paragana—Sarkar—Suba
  2. Sarkar—Paragana—Suba
  3. Suba—Sarkar—Paragana
  4. Paragana—Suba—Sarkar

Ans.(a). Note we are asked the ‘ascending’ order.

A Subah was the term for a province (State) in the Mughal Empire. The governor/ruler of a Subah was known as a subahdar. The subahs were established by emperor Akbar during his administrative reforms of years 1572–1580.

The full Suba was divided into Sarkars (districts), each being looked after by a military officer called the chief Shiqdar. Each Sarkar was divided into smaller units called Parganas. There was a Shiqdar (military officer), Amin or Munsif (Civilian Judge), one Fotahdar (treasurer) and two Karkuns (clerks or writers) in each Pargana.


46.Who among the following was associated as Secretary with Hindu Female School which later came to be known as Bethune Female School?

  1. Annie Besant
  2. Debendranath Tagore
  3. Ishwar Chandra
  4. Vidyasagar Sarojini Naidu

Ans.(c). The college owes its origin to John Elliot Drinkwater Bethune (1801-1851). What began as Hindu Female School in 1849 was renamed as Bethune School in 1856. The Managing Committee of the school was then formed and Pandit Ishwar Chandra Vidyasagar, a supporter of women’s emancipation was made the Secretary.

Ishwar Chandra Vidyasagar (1820 – 1891) was the most prominent campaigner for Hindu widow remarriage, petitioning the Legislative council despite severe opposition, and Lord Dalhousie personally finalised the bill and the Hindu Widows' Remarriage Act, 1856 was passed.


47.In the context of Colonial India, Shah Nawaz Khan, Prem Kumar Sehgal and Gurbaksh Singh Dhillon are remembered as

  1. leaders of Swadeshi and Boycott Movement
  2. members of the Interim Government in 1946
  3. members of the Drafting Committee in the Constituent Assembly
  4. officers of the Indian National Army

Ans.(d). These were INA officers. A very easy question!

Between November 1945 and May 1946, approximately ten courts-martial were held in public at the Red Fort in Delhi. These were called the Red Fort trials, and were a big mistake of the British Raj.

Claude Auchinleck, the Commander-in-Chief of the British-Indian army, hoped that by holding public trials in the Red Fort, public opinion would turn against the INA.

The first and most celebrated joint courts-martial – those of Prem Sahgal, Gurubaksh Singh Dhillon and Shah Nawaz Khan turned out against the Britishers’ expectations. These were officers of the INA, led by Netaji Subhash Bose.


48.With reference to Indian history, which of the following statements is/are correct?

  1. The Nizamat of Arcot emerged out of Hyderabad State.
  2. The Mysore Kingdom emerged out of Vijayanagara Empire.
  3. Rohilkhand Kingdom was formed out of the territories occupied by Ahmad Shah Durrani.

Select the correct answer using the code given below.

  1. 1 and 2
  2. 2 only
  3. 2 and 3
  4. 3 only

Ans.(a). The Kingdom of Rohilkhand arose under the declining Mughal Empire in 1721 and continued to exist until 1774 when annexation by the British transformed its significantly reduced borders into the Princely State of Rampur. Nawab Ali Mohammed Khan, a scion of the ancient Barha Dynasty became the first Nawab of Rohilkhand, having been previously elected as overlord by various Afghan Chiefs at the age of fourteen. He would carve out the future kingdom from the collapsing Mughal Empire and go on to the found the Rohilla Dynasty. Hence, 3 is wrong. Hence options (c) and (d) are wrong.

The Nawabs of the Carnatic (Arcot) ruled the Carnatic region of South India between about 1690 and 1855. It was a dependency of Hyderabad Deccan. This was the time the Mughal Empire gave way to the rising influence of the Maratha Empire, and later the emergence of the British Raj. So 1 is right, so (a) is our answer.


49.Which one of the following statements is correct?

  1. Ajanta Caves lie in the gorge of Waghora river.
  2. Sanchi Stupa lies in the gorge of Chambal river.
  3. Pendulant Cave Shrines lie in the gorge of Narmada river.
  4. Amaravati Stupa lies in the gorge of Godavari river.

Ans.(a). The Ajanta Caves are Buddhist rock-cut cave temples and monasteries, located near Ajanta village, north-central Maharashtra. These are celebrated for their wall paintings. The temples are hollowed out of granite cliffs on the inner side of a 70-foot (20-metre) ravine in the Wagurna River valley 105 km northeast of Aurangabad. So (a) is right.

The Sanchi historic site, west-central Madhya Pradesh state, central India, lies in an upland plateau region, just west of the Betwa River and about 8 km southwest of Vidisha.

The Buddhist monuments Pandavleni Caves also known by the names Pandu Lena caves and Trirashmi caves are a group of 24 rock cut caves. It is close to Nashik city in Maharashtra.

The Britishers were alerted to the ruins of the Stupa at Amaravati after a visit in 1797 by Major Colin Mackenzie. It was on the right bank of the Krishna River in the Andhra district of southeast India.


50.Consider the following statements :

  1. 21st February is declared to be the International Mother Language Day by UNICEF.
  2. The demand that Bangla has to be one of the national languages was raised in the Constituent Assembly of Pakistan.

Which of the above statements is/are correct?

  1. 1 only
  2. 2 only
  3. Both 1 and 2
  4. Neither 1 nor 2

Ans.(a). The idea to celebrate International Mother Language Day was the initiative of Bangladesh. It was approved at the 1999 UNESCO General Conference and has been observed throughout the world since 2000. It is celebrated on 21 Feb each year. So, 1 is correct. So options (b) and (d) are wrong.

Pakistan's Constituent Assembly first convened on August 11, 1947, on the eve of independence and the end of British rule.

In 1948, the then Government of Pakistan declared Urdu to be the sole national language of Pakistan, even though Bengali or Bangla was spoken by the majority of people combining East Pakistan and West Pakistan. The East Pakistan people protested, and demanded that Bangla to be at least one of the national languages, in addition to Urdu. The demand was raised by Dhirendranath Datta from East Pakistan on 23 February 1948, in the constituent Assembly of Pakistan. So, 2 is also correct.


51.How is permaculture farming different from conventional chemical farming?

  1. Permaculture farming discourages monocultural practices but in conventional chemical farming, monoculture practices are pre-dominant.
  2. Conventional chemical farming can cause increase in soil salinity but the occurrence of such phenomenon is not observed in permaculture farming.
  3. Conventional chemical farming is easily possible in semi-arid regions but permaculture farming is not so easily possible in such regions.
  4. Practice of mulching is very important in permaculture farming but not necessarily so in conventional chemical farming.

Select the correct answer using the code given below.

  1. 1 and 3
  2. 1, 2 and 4
  3. 4 only
  4. 2 and 3

Ans.(b). The essence of permaculture is the design of an ecologically sound way of living - in our households, gardens, communities and businesses. It is created by cooperating with nature and caring for the earth and its people.

Permaculture is an approach to land management that adopts arrangements observed in flourishing natural ecosystems. Principles of permaculture discourage monoculture, it opens up the opportunity for growing a wide variety of grains, fruits and vegetables, and widens one’s food basket. Hence statement 1 is true. Only possible options are (a) and (b).

It is proven that the use of pesticides and fertilizers in conventional chemical farming increases the salinity of the soil, but the principles of Permaculture prohibit use of such practices. So statement 2 is also true. Hence, (b) is our answer.


52.With reference to 'palm oil', consider the following statements :

  1. The palm oil tree is native to Southeast Asia.
  2. The palm oil is a raw material for some industries producing lipstick and perfumes.
  3. The palm oil can be used to produce biodiesel.

Which of the statements given above are correct?

  1. 1 and 2 only
  2. 2 and 3 only
  3. 1 and 3 only
  4. 1, 2 and, 3

Ans.(b). Statement 1 is incorrect, as Palm Oil tree is native to Africa but were brought to South-East Asia just over 100 years ago as an ornamental tree crop. Today, Indonesia and Malaysia make up over 85% of global supply but there are 42 other countries that also produce palm oil.

Palm oil is derived from the oil palm tree (Elaeis guineensis Jacq.), native to West Africa and grows best in tropical climates with abundant water.

Three-quarters of total palm oil produced is used for food, particularly cooking oil and processed oils and fats.

Statement 1 is wrong, so only possible option is (b).


53.With reference to the Indus river system, of the following four rivers, three of them pour into one of them which joins the Indus direct. Among the following, which one is such river that joins the Indus direct?

  1. Chenab
  2. Jhelum
  3. Ravi
  4. Sutlej

Ans.(a). An expected question from rivers and tributaries!

Evident that Chenab is the direct tributary of Indus.

Sutlej, Ravi and Jhelum flows into Chenab before meeting Indus.



54.With reference to India, Didwana, Kuchaman, Sargol and Khatu are the names of

  1. glaciers
  2. mangrove areas
  3. Ramsar sites
  4. saline lakes

Ans.(d). All are lakes.

Didwana Lake

Kuchaman Lake – It is a continental saline lake located around Kuchaman city, Nagaur District, Rajasthan. It is 8.5 sq kms in area and is bound by 380 m contour line.

Sargol and Khatu are also saline lakes.


55.Consider the following rivers :

  1. Brahmani
  2. Nagavali
  3. Subarnarekha
  4. Vamsadhara

Which of the above rise from the Eastern Ghats?

  1. 1 and 2
  2. 2 and 4
  3. 3 and 4
  4. 1 and 3

Ans.(b). The Eastern Ghats run from the northern Odisha through Andhra Pradesh to Tamil Nadu in the south passing some parts of Karnataka. They are eroded and cut through by the four major rivers of peninsular India, known as the Godavari, Mahanadi, Krishna, and Kaveri. The mountain ranges run parallel to the Bay of Bengal.

The Eastern Ghats are the source area for many small and medium rivers of the east coastal plains of South India.

Rivers originating on the Eastern Ghats include - Baitarani River, Budhabalanga River, Rushikulya River, Vamsadhara River, Palar River, Nagavali River, Champavathi River, Gosthani River, Sarada River, Sabari River, Sileru River, Tammileru Gundlakamma River, Pennai Yaru River, Swarnamukhi, Kundu River, Vellar River, Penna River.

The Brahmani river is a major seasonal river in Odisha, formed by the confluence of the Sankh and South Koel rivers. Together with the river Baitarani, it forms a large delta before emptying into the Bay of Bengal at Dhamra.


56.Consider the following statements :

  1. The Global Ocean Commission grants licences for seabed exploration and mining in international waters.
  2. India has received licences for seabed mineral exploration in international waters.
  3. 'Rare earth minerals' are present on seafloor in international waters.

Which of the statements given above are correct?

  1. 1 and 2 only
  2. 2 and 3 only
  3. 1 and 3 only
  4. 1, 2 and 3

Ans.(b). The International Seabed Authority (ISA) is an intergovernmental body based in Kingston, Jamaica, that was established to organize, regulate and control all mineral-related activities in the international seabed area beyond the limits of national jurisdiction, an area underlying most of the world's oceans. It is an organization established by the United Nations Convention on the Law of the Sea. So, statement 1 is not correct. So only (b) can be the answer!

(The Global Ocean Commission was a body that existed for a few years since 2013)

In 2017, India’s exclusive rights to explore polymetallic nodules from seabed in Central Indian Ocean Basin (CIOB) have been extended by five years. Vast reserves of vital rare earth minerals are found in ocean bed. So statement 2 and 3 are correct.


57.Among the following, which one is the least water-efficient crop?

  1. Sugarcane
  2. Sunflower
  3. Pearl millet
  4. Red gram

Ans.(a). Around 210 litres of water is required to produce one kg of sugarcane.

Millets are astonishingly low water consuming crops. The rainfall needed for Sorghum, Pearl Millet and Finger Millet is less than 25% of sugarcane and banana, and 30% that of rice.

Sunflower requires far less water than Millets and Sugarcane.

For Red Gram, no irrigation is required during monsoon, but in case of drought or long dry spell during the reproductive period of growth, supplementary/life-saving irrigations should be given. A total of just 20-25 cm of water is sufficient to raise a good crop.


58.Consider the following statements :

  1. In the tropical zone, the western sections of the oceans are warmer than the eastern sections owing to the influence of trade winds.
  2. In the temperate zone, westerlies make the eastern sections of oceans warmer than the western sections.

Which of the statements given above is/are correct?

  1. 1 only
  2. 2 only
  3. Both 1 and 2
  4. Neither 1 nor 2

Ans.(c). Air temperature differences across the Earth's surface (both land and water) create winds, with warm air being lighter than cold air. Near the equator, the sun heats the sea surface, causing the warm air at the surface to rise and be replaced by the trade winds blowing from subtropical high pressure systems into equatorial low-pressure troughs. The trade winds blow steadily for days and are among the most consistent on earth. When trade winds move over warm tropical waters, they pick up moisture and bring heavy rainfall to the windward-facing slopes of mountainous areas, contrasting with the downward motion of dry air that creates desert areas on land. Because the area of Earth between the Tropic of Cancer and Tropic of Capricorn, lying at approximately 23 degrees latitude on either side of the equator, receives more solar heat than the rest of the earth, the warm air creates clouds and rain with thunder-showers there almost every day.

The influence of the trade winds on weather and climate is seen with El Niño, La Niña, and the development of hurricanes/cyclones. The differences in pressure and temperature between the two sides of the Pacific are caused by the trade winds; air blowing from east to west pushes water, making the sea level higher in the western Pacific, and makes cold water rise toward the surface, making the eastern Pacific approximately 14 degrees F (7.7 degrees C) cooler than the western Pacific.

So statement 1 is correct.

In the mid-latitudes, the gyre circulations move poleward along the eastern coasts of continents (the western edges of the ocean), bringing warm water. The water off the east coasts of these places is thus warmer than one expects for that latitude. The return flow on the eastern edges of ocean basins (west coast of continents) brings cold water from poleward of that latitude. This analysis applies to the mid-latitudes only. There are opposite gyres closer to the poles that make the east coasts much colder than expected. (Compare the Alaska current to the Labrador, which is off the east coast of Canada.)

So statement 2 is correct.


59.In the context of India's preparation for Climate-Smart Agriculture, consider the following statements :

  1. The 'Climate-Smart Village' approach in India is a part of a project led by the Climate Change, Agriculture and Food Security (CCAFS), an international research programme.
  2. The project of CCAFS is carried out under Consultative Group on International Agricultural Research (CGIAR) headquartered in France.
  3. The International Crops Research Institute for the Semi-Arid Tropics (ICRISAT) in India is one of the CGIAR's research centres.

Which of the statements given above are correct?

  1. 1 and 2 only
  2. 2 and 3 only
  3. 1 and 3 only
  4. 1, 2 and 3

Ans.(d). CCAFS’s major activities in India include test, evaluate and develop portfolios of climate-smart interventions for different agro-ecological zones and farm types; promotion of CSA through the Climate-Smart Village (CSV) approach; weather-based insurance; use of ICT for dissemination of climate information based agro-advisories; mapping hotspots of germplasm collection and conservation; crowdsourcing farmers preferences for stress resistant varieties; gender and social inclusion in climate change adaptation; and development of decision-support tools for planning and investment in adaptation and mitigation activities. CCAFS and CGIAR centers together are engaging with national and sub-national stakeholders including policymakers, NGOs and civil society groups, research organizations, farmers groups and private sector for promotion of climate-smart agriculture in India. Priority areas include Indo-Gangetic Plan of Northern India and dryland agriculture systems in southern and western parts of India. CGIAR is a global partnership that unites international organizations engaged in research about food security. It is Headquarterd at Montpellier, France.

ICRISAT, a CGIAR Research Center, is a non-profit, non-political public international research organization that conducts agricultural research for development in Asia and sub-Saharan Africa with a wide array of partners throughout the world, headquartered in Patancheru (Telangana), India with several regional centers.

Hence all the three statements are correct.


60."Leaf litter decomposes faster than in any other biome and as a result the soil surface is often almost bare. Apart from trees, the vegetation is largely composed of plant forms that reach up into the canopy vicariously, by climbing the trees or growing as epiphytes, rooted on the upper branches of trees." This is the most likely description of

  1. coniferous forest
  2. dry deciduous forest
  3. mangrove forest
  4. tropical rain forest

Ans.(d). Leaf litter decomposition in terrestrial ecosystems has a major role in the biogeochemical cycling of elements in the environment. Climatic features, like temperature, rainfall, humidity, and seasonal variations affect the rate of litter decomposition.

The tropical rainforest is a hot, moist biome where it rains all year long. It is known for its dense canopies of vegetation that form three different layers. The top layer or canopy contains giant trees that grow to heights of 75 m (about 250 ft) or more. This layer of vegetation prevents much of the sunlight from reaching the ground. Thick, woody vines are also found in the canopy. They climb trees in the canopy to reach for sunlight. The middle layer, or understory, is made up of vines, smaller trees, ferns, and palms.

The hot, humid climate leads to rapid chemical weathering of the bedrock. In addition to this the climate provides ideal conditions for the rapid decomposition of leaf litter. However, the soils are fragile and depend upon the rapid decomposition of leaf litter for nutrients. The rapid decomposition leads to a thin humus layer. If vegetation is removed nutrients are quickly leached and the soil rapidly loses fertility.


61.The vegetation of savannah consists of grassland with scattered small trees, but extensive areas have no trees. The forest development in such areas is generally kept in check by one or more or a combination of some conditions. Which of the following are such conditions?

  1. Burrowing animals and termites
  2. Fire
  3. Grazing herbivores
  4. Seasonal rainfall
  5. Soil properties

Select the correct answer using the code given below.

  1. 1 and 2
  2. 4 and 5
  3. 2, 3 and 4
  4. 1, 3 and 5

Ans.(c). Savannas are defined based on vegetation structure, the central idea being a discontinuous tree cover in a continuous grass understorey. Fire, grazing herbivore and seasonal rainfall are behind the scarce forest development in the region.

The tree component of savannas generally becomes more important as rainfall increases, but other factors such as topography, soil, and grazing intensity all exert influences in complex and variable ways. Dry-season fires, fueled by dried grass, may kill some trees, especially the more-vulnerable young saplings, and, therefore, their severity also greatly affects the nature of savanna vegetation. So "fire" is correct, and 2 must be in the answer.

So options (b) and (d) are gone.

Clearly 3 and 4 are important.

Savannas get very little rain – about 4 inches (100 mm) of rain – in the dry season, and they will often not get any rainfall at all for many months. This is a long time for plants to go without water. But the wet season gets lots of rain.

(An unusually large proportion of dead organic matter—approximately 30 percent—is decomposed through the feeding activities of termites. Thus, a significant proportion of released mineral nutrients may be stored for long periods in termite mounds where they are not readily available to plant roots. In savannas in Thailand it has been shown that soil fertility can be markedly improved by mechanically breaking up termite mounds and spreading the material across the soil surface. In Kenya, old termite mounds, which are raised above the general soil surface, also provide flood-proof sites where trees and shrubs can grow, with grassland between them, forming the so-called termite savanna.)


62.With reference to the water on the planet Earth, consider the following statements :

  1. The amount of water in the rivers and lakes is more than the amount of groundwater.
  2. The amount of water in polar ice caps and glaciers is more than the amount of groundwater.

Which of the statements given above is/are correct?

  1. 1 only
  2. 2 only
  3. Both 1 and 2
  4. Neither 1 nor 2

Ans.(b). It is evident from the table below that only statement 2 is correct.



63.Consider the following statements :

  1. Moringa (drumstick tree) is a leguminous evergreen tree.
  2. Tamarind tree is endemic to South Asia.
  3. In India, most of the tamarind is collected as minor forest produce.
  4. India exports tamarind and seeds of moringa.
  5. Seeds of moringa and tamarind can be used in the production of biofuels.

Which of the statements given above are correct?

  1. 1, 2, 4 and 5
  2. 3, 4 and 5
  3. 1, 3 and 4
  4. 1, 2, 3 and 5

Ans.(b). Moringa Oleifera (Drumstick tree) is the best-known species of moringa tree family Moringaceae. It is the fast-growing, drought resistance tree, native to the sub-Himalayan tract of north India, Pakistan, Bangladesh, and Afghanistan. It is considered to have an origin in Agra and Oudh, the northwest of India. The tree is rich in phytosterol like stigmasterol, sitosterol, and campesterol which are the precursor for hormones. It is a non-leguminous tree. So, 1 is not correct.

(Crops which have nodules in their roots which contain nitrogen fixing bacteria are called leguminous crops.)

If 1 is not correct, then only (b) can be our answer!

Tamarind, (Tamarindus indica), evergreen tree of the pea family (Fabaceae), native to tropical Africa. So statement 2 is also incorrect.


64.The black cotton soil of India has been formed due to the weathering of

  1. brown forest soil
  2. fissure volcanic rock
  3. granite and schist
  4. shale and limestone

Ans.(b). Among the in-situ soils of India, the black soils found in the lava-covered areas are the most visible ones. Those are often popularly known as “black cotton soils,” since cotton has been the most common traditional crop in areas where they are found.

Black soils are derivatives of trap lava and are spread mostly across interior Gujarat, Maharashtra, Karnataka, and Madhya Pradesh on the Deccan lava plateau and the Malwa Plateau, where there is both moderate rainfall and underlying basaltic rock.

Black soil is a result of weathering of igneous basaltic rocks, a type of extrusive rock. Black soil is generally found in the Deccan trap, formed 66 million years ago. It is generally 2000 meters in depth.


65.With reference to recent developments regarding 'Recombinant Vector Vaccines', consider the following statements :

  1. Genetic engineering is applied in the development of these vaccines.
  2. Bacteria and viruses are used as vectors.

Which of the statements is/are correct?

  1. 1 only
  2. 2 only
  3. Both 1 and 2
  4. Neither 1 nor 2

Ans.(c). Genetic engineering is broad term that refers to the process that is used to manipulate the genetic structure of an organism. Recombinant technology is one of the methods of Genetic Engineering. So clearly, 1 is correct. So options (b) and (d) are wrong.

Though Recombinant Vector Vaccines are primarily developed using virus, they can also be developed using bacteria also. Hence 2 is also correct.


66.In the context of hereditary diseases, consider the following statements :

  1. Passing on mitochondrial diseases from parent to child can be prevented by mitochondrial replacement therapy either before or after in vitro fertilization of egg.
  2. A child inherits mitochondrial diseases entirely from mother and not from father.

Which of the statements given above is/are correct?

  1. 1 only
  2. 2 only
  3. Both 1 and 2
  4. Neither 1 nor 2

Ans.(a). Mitochondrial diseases are chronic (long-term), genetic, often inherited disorders that occur when mitochondria fail to produce enough energy for the body to function properly. (Inherited means the disorder was passed on from parents to children.) Mitochondrial diseases can be present at birth, but can also occur at any age.

Primary mitochondrial disease is a genetic condition that can be inherited (passed from parents to their children) in several ways. Under normal circumstances, a child inherits genes in pairs -- one gene from the mother and one from the father. A child with a mitochondrial disease does NOT receive a normal pair of genes from the parents (but mutated ones).

Inheritance types are many – (i) Autosomal recessive inheritance: This child receives one mutated copy of a gene from each parent. There is a 25% chance that each child in the family will inherit a mitochondrial disease; (ii) Autosomal dominant inheritance: This child receives one mutated copy of a gene from either parent. There is a 50% chance that each child in the family will inherit a mitochondrial disease; (iii) Mitochondrial inheritance: In this unique type of inheritance, the mitochondria contain their own DNA. Only mitochondrial disorders caused by mutations in the mitochondrial DNA are exclusively inherited from mothers. If this is the way a mitochondrial disease was inherited, there is a 100% chance that each child in the family will inherit a mitochondrial disease; and (iv) Random mutations: Sometimes genes develop a mutation of their own that is not inherited from a parent.

Thus it is only in “Mitochondrial inheritance” that disease comes from mother and not father. So 2 is not right. So (b) and (c) are ruled out.

MRT is possible after and before in vitro fertilization.

Pronuclear transfer technique (PTT) – It is a significant approach of MRT administered after fertilization, in which two zygotes are raised in vitro. One of the zygotes belongs to the biological parents with pronuclei and defective mitochondria and the other one to the donor with pronuclei and healthy mitochondria

Maternal spindle transfer (MST) technique – The technique, executed before fertilization is a form of selective reproduction similar to prenatal diagnosis and pre-implantation genetic diagnosis (PGD).

So 1 is right and (a) is the answer.


67.Bollgard I and Bollgard II technologies are mentioned in the context of

  1. clonal propagation of crop plants
  2. developing genetically modified crop plants
  3. production of plant growth substances
  4. production of biofertilizers

Ans.(b). Bollgard I, the predominantly cultivated pest tolerant cotton, contains only one gene, the Cry 1Ac, from Bacillus thuringiensis.

Bollgard II contains the Cry 2 Ab gene, in addition to Cry 1 Ac. The Bollgard II event developed by Mahyco-Monsanto Biotech Ltd, (MMBL) is technically designated as MON 15985.


68.In a pressure cooker, the temperature at which the food is cooked depends mainly upon which of the following?

  1. Area of the hole in the lid
  2. Temperature of the flame
  3. Weight of the lid

Select the correct answer using the code given below.

  1. 1 and 2 only
  2. 2 and 3 only
  3. 1 and 3 only
  4. 1, 2 and 3

Ans.(c). 1 and 3 only.

The simple logic of pressure cooking is that if you raise the pressure in an enclosed space, then the boiling point of liquid contained in that space will also rise. Water has a boiling point of 100° C at 1 atmosphere. If pressure rises, water’s boiling point rises too. It carries more heat, and cook things faster.

A pressure cooker has a modified lid that locks on over a rubber gasket to create a seal. The cooker works by raising the temperature of boiling water. To use a pressure cooker, we put the food in the pot with some liquid. Once the lid is locked in place and the cooker is set on high heat, steam develops in the pot and can’t escape. The trapped steam increases the atmospheric pressure inside the cooker above normal sea-level pressure. At that pressure, the boiling point of water is increased. This higher temperature is what cooks food faster. Once the cooker has reached full pressure, usually indicated by a gauge or pop-up rod on the lid, a release valve opens, letting out steam in a regulated flow to maintain a constant temperature inside the pot.

More will be the weight of the lid, more pressure and temperature the cooker can sustain. The area of the hole in the lid should be appropriate; if it is larger then proper pressure and temperature will not be maintained.

But the temperature of the flame will not matter, as it will decide the “time” taken to cook the food, nothing else. A higher temperature will cook the food faster, and vice versa. It will not affect the pressure inside the cooker because even a low temperature flame, given sufficient time, will build the needed pressure anyway.


69.Consider the following :

  1. Bacteria
  2. Fungi
  3. Virus

Which of the above can be cultured in artificial/ synthetic medium?

  1. 1 and 2 only
  2. 2 and 3 only
  3. 1 and 3 only
  4. 1, 2 and 3

Ans.(a). Viruses can infect animals, plants, and even other microorganisms. Since viruses lack metabolic machinery of their own and are totally dependent on their host cell for replication, they cannot be grown in synthetic culture media. Only option possible is option (a).


70.Consider the following statements :

  1. Adenoviruses have single-stranded DNA genomes whereas retroviruses have double-stranded DNA genomes.
  2. Common cold is sometime caused by an adenovirus whereas AIDS is caused by a retrovirus. Which of the statements given above is/are correct?
  1. 1 only
  2. 2 only
  3. Both 1 and 2
  4. Neither 1 nor 2

Ans.(b). Adenoviruses are medium-sized, nonenveloped viruses with an icosahedral nucleocapsid containing a double stranded DNA, whereas retroviruses are single-stranded RNA animal viruses that employ a double-stranded DNA intermediate.

An adenovirus is considered as the largest among the non-enveloped viruses. This term indicates that the virus has no protective protein-coating, called capsids, to wrap the genetic information stored inside the virus. This virus contains a double-stranded DNA. It has been proven that this virus is the cause for about 10% of upper respiratory infections in children and adults. Adenoviruses are common viruses that cause a range of illness. They can cause cold-like symptoms, fever, sore throat, bronchitis, pneumonia, diarrhea, and pink eye (conjunctivitis).

A retrovirus, on the other hand, is an example of an enveloped virus. In this case, it has a protective protein coat that makes it more resilient and has a higher tendency to cause diseases. It is considered as an RNA virus and can integrate itself to its host cells, thus, causing damage to the cell. The disease AIDS is caused by a retrovirus named HIV.

So, statement 1 is incorrect and statement 2 is correct.


71.Water can dissolve more substances than any other liquid because

  1. it is dipolar in nature
  2. it is a good conductor of heat
  3. it has high value of specific heat
  4. it is an oxide of hydrogen

Ans.(a). Water is called the "universal solvent" because it is capable of dissolving more substances than any other liquid. It is water's chemical composition and physical attributes that make it such an excellent solvent.

Water molecules have a polar arrangement of oxygen and hydrogen atoms—one side (hydrogen) has a positive electrical charge and the other side (oxygen) had a negative charge.

This phenomenon is called the dipolar nature of the water molecule.

This allows the water molecule to become attracted to many other different types of molecules.


72.With reference to street-lighting, how do sodium lamps differ from LED lamps?

  1. Sodium lamps produce light in 360 degrees but it is not so in the case of LED lamps.
  2. As street-lights, sodium lamps have longer life span than LED lamps.
  3. The spectrum of visible light from sodium lamps is almost monochromatic while LED lamps offer significant colour advantages in street-lighting.

Select the correct answer using the code given below.

  1. 3 only
  2. 2 only
  3. 1 and 3 only
  4. 1,2 and 3

Ans.(c). LED lamps provide directional light (over 180 degrees), unlike Sodium lamps with 360 degrees light direction. Hence statement 1 is true. Answer can be (c) or (d).

LED lamps as street light has longest lifespan compared with other street light types (50,000 to 100,000 hours). Hence statement 2 is not true. Answer is option (c).

Color Rendering Index (CRI) - CRI is how the light source reflects the color of an object, showing its aesthetics and form. Criteria for Street Lights – 75 to 100: Excellent; 65 to 75: Good; 55 to 65: Fair; 0 to 55: Poor. LED street lights have CRI of 65 to 95 while Sodium lamps have CRI of 20 to 30 (almost monochromatic). Hence statement 3 is correct.


73.The term ACE2' is talked about in the context of

  1. genes introduced in the genetically modified plants
  2. development of India's own satellite navigation system
  3. radio collars for wildlife tracking
  4. spread of viral diseases

Ans.(d). ACE2 stands for Angiotensin-converting enzyme 2. It has been in news continuously due to Covid-19.

The ACE2 is a protein on the surface of many cell types. It is an enzyme that generates small proteins – by cutting up the larger protein angiotensinogen – that then go on to regulate functions in the cell. Using the spike-like protein on its surface, the SARS-CoV-2 virus binds to ACE2 – like a key being inserted into a lock – prior to entry and infection of cells. Hence, ACE2 acts as a cellular doorway – a receptor – for the virus that causes COVID-19.


74.Bisphenol A (BPA), a cause of concern, is a structural/key component in the manufacture of which of the following kinds of plastics?

  1. Low-density polyethylene
  2. Polycarbonate
  3. Polyethylene terephthalate
  4. Polyvinyl chloride

Ans.(b). Bisphenol A (BPA) is a chemical produced in large quantities for use primarily in the production of polycarbonate plastics and epoxy resins.

Exposure to BPA is a concern because of the possible health effects on the brain and prostate gland of fetuses, infants and children. It can also affect children's behaviour. Research suggests a possible link between BPA and increased blood pressure, type 2 diabetes and cardiovascular disease.


75.`Triclosan', considered harmful when exposed to high levels for a long time, is most likely present in which of the following

  1. Food preservatives
  2. Fruit-ripening substances
  3. Reused plastic containers
  4. Toiletries

Ans.(d). Triclosan is an ingredient added to many consumer products intended to reduce or prevent bacterial contamination. It is added to some antibacterial soaps and body washes, toothpastes, and some cosmetics—products regulated by the U.S. Food and Drug Administration (FDA). It also can be found in clothing, kitchenware, furniture, and toys—products not regulated by the FDA.

Some animal studies have shown that exposure to high doses of triclosan is associated with a decrease in the levels of some thyroid hormones. There are other ongoing studies that involve the safety of triclosan. One is a study investigating the potential of developing skin cancer after a long-term exposure to triclosan in animals.


76.Which one of the following is a reason why astronomical distances are measured in light-years?

  1. Distances among stellar bodies do not change.
  2. Gravity of stellar bodies does not change
  3. Light always travels in straight line.
  4. Speed of light is always same.

Ans.(d). An excellent question.

A light-year is how astronomers measure distance in space. It’s defined by how far a beam of light travels in one year – a distance of six trillion miles.

Reasons to measure astronomical distances in light year are – (a) Speed of light is constant, and (b) It is large enough for such lengths and is even concise.

Measuring in light-years also allows astronomers to determine how far back in time they are viewing. Because light takes time to travel to our eyes, everything we view in the night sky has already happened. In other words, when you observe something 1 light-year away, you see it as it appeared exactly one year ago.


77.We adopted parliamentary democracy based on the British model, but how does our model differ from that model?

  1. As regards legislation, the British Parliament is supreme or sovereign but in India, the power of the Parliament to legislate is limited.
  2. In India, matters related to the constitutionality of the Amendment of an Act of the Parliament are referred to the Constitution Bench by the Supreme Court.

Select the correct answer using the code given below.

  1. 1 only
  2. 2 only
  3. Both 1 and 2
  4. Neither 1 nor 2

Ans.(c). The parliamentary system of India is largely based on the British parliamentary system but it never became a replica of the British system and differs in the following respects –

  1. India has a republican system in place of British monarchical system
  2. The British system is based on the doctrine of the sovereignty of Parliament, while the Parliament is not supreme in India and enjoys limited and restricted powers due to a written Constitution, federal system, judicial review and fundamental rights.

Hence statement 1 is true.

Now, a Constitution Bench is a bench of the Supreme Court having five or more judges on it. These are not a routine phenomenon. A vast majority of cases before the Supreme Court are heard and decided by a bench of two judges (called a Division Bench), and sometimes of three. Constitution Benches are exceptions, set up only if one or more of the following circumstances exist:

  1. The case involves a substantial question of law pertaining to the interpretation of the Constitution;
  2. The President of India has sought the Supreme Court’s opinion on a question of fact or law under Article 143 of the Constitution;
  3. Two or more three-judge benches of the Supreme Court have delivered conflicting judgments on the same point of law, thus warranting a definitive pronouncement by a larger bench;
  4. A later three-judge bench doubts the correctness of a judgment delivered by a previous three-judge bench of the Supreme Court, and decides to refer the case to a larger bench for a reconsideration of that earlier judgment.

So statement 2 is true whenever it happens (but in UK, it cannot happen at all).

Hence answer is (c).


78.With reference to the Union Government, consider the following statements :

  1. N. Gopalaswamy lyengar Committee suggested that a minister and a secretary be designated solely for pursuing the subject of administrative reform and promoting it.
  2. In 1970, the Department of Personnel was constituted on the recommendation of the Administrative Reforms Commission, 1966, and this was placed under the Prime Minister's charge.

Which of the statements given above is/are correct?

  1. 1 only
  2. 2 only
  3. Both 1 and 2
  4. Neither 1 nor 2

Ans.(d). N G Iyengar was part of the 7-member drafting committee of the Indian constitution appointed on August 29, 1947. Sh. N. Gopalaswamy Ayyangar, in his Report namely ‘Reorganization of the Machinery of Central Government’ in 1950 recommended for the grouping of ministries, improvement in the capabilities of the personnel, and also in the working of the O&M Division.

So statement 1 is incorrect.

On 07th February, 1973, the work relating to Department of Administrative Reforms was transferred to Department of Personnel created under Cabinet Secretariat on 01st August, 1970 and was re-designated as Department of Personnel and Administrative Reforms.

( refer https://darpg.gov.in/about-department-0 )

So statement 2 is not correct.

Hence both statements 1 and 2 are not correct.


79.'Right to Privacy' is protected under which Article of the Constitution of India?

  1. Article 15
  2. Article 19
  3. Article 21
  4. Article 29

Ans.(c). In its landmark verdict in the Puttaswamy case, the Right to Privacy was declared a fundamental right by the Supreme Court of India. This right is protected as an intrinsic part of the right to life and personal liberty under Article 21 and as a part of the freedoms guaranteed by Part III of the Constitution.

Article 21 of Constitution of India: Protection of Life and Personal Liberty. Article 21 states that “No person shall be deprived of his life or personal liberty except according to a procedure established by law.” Thus, article 21 secures two rights: Right to life, and Right to personal liberty.

Article 15 of the Constitution of India forbids discrimination on grounds only of religion, race, caste, sex, or place of birth. So option (a) is not correct.

Article 19 of the Constitution provides freedom of speech which is the right to express one's opinion freely without any fear through oral/written/electronic/broadcasting/press.

Article 29 of the Indian Constitution is related with Protection of interests of minorities.


80.Consider the following statements :

  1. In India, there is no law restricting the candidates from contesting in one Lok Sabha election from three constituencies.
  2. In 1991 Lok Sabha Election, Shri Devi Lal contested from three Lok Sabha constituencies.
  3. As per the existing rules, if a candidate contests in one Lok Sabha election from many constituencies, his/her party should bear the cost of bye-elections to the constituencies vacated by him/her in the event of him/her winning in all the constituencies.

Which of the statements given above is/are correct?

  1. 1 only
  2. 2 only
  3. 1 and 3
  4. 2 and 3

Ans.(b). Elections in India are governed by the Representation of People’s Act, 1951 (RPA), and conducted and organised by the Election Commission of India (ECI).

As per Section 33(7) of the RPA, one candidate can contest from a maximum of two constituencies (more constituencies were allowed until 1996 when the RPA was amended to set the cap at two constituencies).

So, statement 1 is incorrect. Answer could be option (b) or (d).

The Election Commission recommended amending Section 33(7) so as to allow one candidate to contest from only one seat. It did so in 2004, 2010, 2016 and again in 2018. These were recommendations given to the Executive and the Supreme Court. (If not an amendment, the EC opined that a system should be devised wherein if a candidate contested from two constituencies and won both, then he or she would bear the financial burden of conducting the subsequent by-election in one of the constituencies. The amount would be Rs 5 lakh for a Vidhan Sabha election and Rs 10 lakh for a Lok Sabha election.)

So, statement 3 is not correct.

So answer should be (b). [but we were not able to confirm which 3 seats did Mr Devi Lal contest from, in the 1991 LS elections]


81.Consider the following statements :

  1. 'Right to the City' is an agreed human right and the UN-Habitat monitors the commitments made by each country in this regard.
  2. 'Right to the City' gives every occupant of the city the right to reclaim public spaces and public participation in the city.
  3. 'Right to the City' means that the State cannot deny any public service or facility to the unauthorized colonies in the city.

Which of the statements given above is/are correct?

  1. 1 only
  2. 3 only
  3. 1 and 2
  4. 2 and 3

Ans.(c). Refer site https://www.right2city.org/the-right-to-the-city/

The Right to the City is the right of all inhabitants, present and future, permanent and temporary, to inhabit, use, occupy, produce, govern and enjoy just, inclusive, safe and sustainable cities, villages and human settlements, defined as commons essential to a full and decent life.

So statement 2 is correct. So options (a) and (b) are ruled out.

The Right to the City is a collective right that highlights the territorial integrality and interdependence of all internationally recognized civil, political, economic, social, cultural and environmental rights, as regulated in international human rights treaties, bringing them the territorial dimension and a focus on adequate life standards.

Now refer to - https://uploads.habitat3.org/hb3/Habitat%20III%20Policy%20Paper%201.pdf

So statement 1 seems right.

Hence answer is (c).


82.With reference to India, consider the following statements :

  1. Judicial custody means an accused is in the custody of the concerned magistrate and such accused is locked up in police station, not in jail.
  2. During judicial custody, the police officer in charge of the case is not allowed to interrogate the suspect without the approval of the court.

Which of the statements given above is/are correct?

  1. 1 only
  2. 2 only
  3. Both 1 and 2
  4. Neither 1 nor 2

Ans.(b). The word 'custody' means apprehending someone for protective care.

Police Custody (PC) means that police has the physical custody of the accused while Judicial Custody (JC) means an accused is in the custody of the concerned Magistrate.

In former, the accused is lodged in police station lockup while in latter, it is the jail.

During Judicial Custody, the police officer in charge of the case is not allowed to interrogate the suspect. However, the court may allow the interrogations to be conducted if it opines the interrogation being necessary under the facts produced before the court.

Hence only statement 2 is correct.


83.With reference to India, consider the following statements :

  1. When a prisoner makes out a sufficient case, parole cannot be denied to such prisoner because it becomes a matter of his/her right.
  2. State Governments have their own Prisoners Release on Parole Rules.

Which of the statements given above is/are correct?

  1. 1 only
  2. 2 only
  3. Both 1 and 2
  4. Neither 1 nor 2

Ans.(b). A Parole in Criminal Law is the release of a convict from imprisonment upon certain conditions to be observed by him. It is a release from prison after part of the sentence has been served, the prisoner still remaining in custody and under stated conditions until discharged and liable to return to the institution for violation of any of these conditions.

The power to grant Parole is essentially the function of the executive and to consider the request made by the prisoner for his release on parole is the prerogative of the executive only.

Rules have been framed in every State regulating the grant of parole. Hence statement 2 is true.

Every prisoner before his entitlement for his release on parole has to meet certain eligibility criteria.

Further in the following cases, parole would ordinarily be not granted except, if in the discretion of the competent authority special circumstances exist for grant of parole;

  1. If the prisoner is convicted of murder after rape;
  2. If the prisoner is convicted for murder and rape of children;
  3. If prisoner is convicted for multiple murders.

Hence statement 1 is not true.


84.At the national level, which ministry is the nodal agency to ensure effective implementation of the Scheduled Tribes and Other Traditional Forest Dwellers (Recognition of Forest Rights) Act, 2006?

  1. Ministry of Environment, Forest and Climate Change
  2. Ministry of Panchayati Raj
  3. Ministry of Rural Development
  4. Ministry of Tribal Affairs

Ans.(d). The Scheduled Tribes And Other Traditional Forest Dwellers (Recognition Of Forest Rights) Act, 2006 - The Forest Rights Act (FRA), 2006 - recognizes the rights of the forest dwelling tribal communities and other traditional forest dwellers to forest resources, on which these communities were dependent for a variety of needs, including livelihood, habitation and other socio-cultural needs.

It is explicitly mentioned in the Chapter VI of the act is that “Nodal agency – The Ministry of the Central Government dealing with Tribal Affairs or any officer or authority authorised by the Central Government in this behalf shall be the nodal agency for the implementation of the provisions of this Act”.


85. A legislation which confers on the executive or administrative authority an unguided and uncontrolled discretionary power in the matter of application of law violates which one of the following Articles of the Constitution of India?

  1. Article 14
  2. Article 28
  3. Article 32
  4. Article 44

Ans.(a). Apart from preamble Article 14 of Indian Constitution ensures equality before law and equal protection of law to the citizen of India.

Article 14 which at the root of arbitrariness and Article 21 guarantees right to life and liberty which is the fundamental provision to protect liberty and ensure life with dignity. It bars discrimination and prohibits both discriminatory laws and administrative action.

Article 22 guarantees natural justice and provision of fair hearing to the arrested person.

Article 28 deals with Freedom as to attendance at religious instruction or religious worship in certain educational institutions.

Article 32 deals with the 'Right to Constitutional Remedies', or affirms the right to move the Supreme Court by appropriate proceedings for the enforcement of the rights conferred in Part III of the Constitution.

Article 44 of Indian Constitution is related with Uniform Civil Code.


86. Which one of the following in Indian polity is an essential feature that indicates that it is federal in character?

  1. The independence of judiciary is safeguarded.
  2. The Union Legislature has elected representatives from constituent units.
  3. The Union Cabinet can elected representatives regional parties.
  4. The Fundamental Rights are enforceable by Courts of Law.

Ans.(a). Constitutions are either unitary or federal. In the unitary government, the powers of the government are centralised in the central government and the states are subordinate to the centre.

In a federal constitution, there is a division of power between the states and the central government and both are independent in their own spheres.

Indian Constitution is neither purely federal nor purely integral. It is a harmonious mixture of both.

Key features of federalism can be listed as

  1. There are two or more levels (or tiers) of government
  2. Different tiers of government govern the same citizens, but each tier has its own jurisdiction in specific matters of legislation, taxation and administration.
  3. The jurisdictions of the respective levels or tiers of government are specified in the constitution. So the existence and authority of each tier of government is constitutionally guaranteed.
  4. The fundamental provisions of the constitution cannot be unilaterally changed by one level of government. Such changes require the consent of both the levels of government.
  5. Courts have the power to interpret the constitution and the powers of different levels of government. The highest court acts as an umpire if disputes arise between different levels of government in the exercise of their respective powers.
  6. The most important feature of the federal system adopted by the Indian Constitution is the principle that relations between the States and the centre would be based on cooperation. For this, an independent Judiciary is a must.
  7. Hence, Option (a) is correct.


87.Which one of the following best defines the term 'State'?

  1. A community of persons permanently occupying a definite territory independent of external control and possessing an organized government
  2. A politically organized people of a definite territory and possessing an authority to govern them, maintain law and order, protect their natural rights and safeguard their means of sustenance
  3. A number of persons who have been living in a definite territory for a very long time with their own culture, tradition and government
  4. A society permanently living in a definite territory with a central authority, an executive responsible to the central authority and an independent judiciary

Ans.(a). The term ‘State’ is central to the study of Political Science. But it is wrongly used as synonym for nation, society, government etc.

Some of the definitions of the concept of State are as follows:

“The State is the politically organized people of a definite territory” - Bluntschli

State is “a community of persons, more or less numerous, permanently occupying a definite portion of territory, independent, or nearly so, of external control, and possessing an organized government to which the great body of inhabitants render habitual obedience.” – Garner

State is “a territorial society divided into governments and subjects, whether individuals or associations of individuals, whose relationships are determined by the exercise of this supreme coercive power.” – Laski

State “is a people organized for law within a definite territory”. –Woodrow Wilson

“The State is a concept of political science, and a moral reality which exists where a number of people, living on a definite territory, are unified under a government which in internal matters is the organ of expressing their sovereignty, and in external matters is independent of other governments.” – Gilchrist

A ”state” is such that it alone has the prerogative of making laws. The lawmaking power derives from sovereignty, which is the most distinctive characteristic of the State.

Only option (a) suits the definitions.


88.With reference to Indian judiciary, consider the following statements :

  1. Any retired judge of the Supreme Court of India can be called back to sit and act as a Supreme Court judge by the Chief Justice of India with prior permission of the President of India.
  2. A High Court in India has the power to review its own judgement as the Supreme Court does.

Which of the statements given above is/are correct?

  1. 1 only
  2. 2 only
  3. Both 1 and 2
  4. Neither 1 nor 2

Ans.(c). IStatement 1 is correct: Under Article 128 of the Constitution, the Chief Justice of India may, at any time, with the previous consent of the President, request any person who has held the office of a Judge of the Supreme Court to sit and act as a Judge of the Supreme Court.

Statement 2 is correct: Being “A Court Of Record”, a High Court can review its own judgments under Article 226 Of the Constitution. The Kerala High Court has iterated the legal proposition that High Courts as Courts of Record could review their own orders. A Division Bench of Chief Justice S Manikumar and Shaji P Chaly were confronted with an appeal against a review petition.


89.With reference to India, consider the following statements :

  1. There is only one citizenship and one domicile.
  2. A citizen by birth only can become the Head of State.
  3. A foreigner once granted the citizenship cannot be deprived of it under any circumstances.

Which of the statements given above is/are correct?

  1. 1 only
  2. 2 only
  3. 1 and 3
  4. 2 and 3

Ans.(a). Part II of the Constitution of India (Articles 5-11) deals with the Citizenship of India.

ARTICLE 5: CITIZENSHIP AT THE COMMENCEMENT OF THE CONSTITUTION

At the commencement of this Constitution, every person who has his domicile in the territory of India and –

  1. who was born in the territory of India; or
  2. either of whose parents was born in the territory of India; or
  3. who has been ordinarily resident in the territory of India for not less than five years immediately preceding such commencement, shall be a citizen of India.

ARTICLE 9: PERSONS VOLUNTARILY ACQUIRING CITIZENSHIP OF A FOREIGN STATE NOT TO BE CITIZENS

No person shall be a citizen of India by virtue of article 5 or be deemed to be a citizen of India by virtue of article 6 or article 8 if he has voluntarily acquired the citizenship of any foreign State.

Hence, statement 1 is correct.

ARTICLE 10: CONTINUANCE OF THE RIGHTS OF CITIZENSHIP

Every person who is or is deemed to be a citizen of India under any of the foregoing provisions of this Part shall, subject to the provisions of any law that may be made by Parliament, continue to be such citizen.

Deprivation Of Citizenship [Section 10 or Indian Citizenship Act, 1955]

Deprivation is a compulsory termination of citizenship of India. A citizen of India by naturalization, registration, domicile and residence, may be deprived of his citizenship by an order of the Central Government if it is satisfied that:

The citizen has obtained the citizenship by means of fraud, false representation or concealment of any material fact;

The citizen has shown disloyalty to the Constitution of India;

The citizen has unlawfully traded or communicated with the enemy during a war;

The citizen has, within five years after registration or neutralization, been imprisoned in any country for two years;

The citizen has been ordinarily resident out of India for seven years continuously.

Hence, Statement 3 is not correct.

Article 58 of the constitution sets the principal qualifications one must meet to be eligible to the office of the president. A president must be: a citizen of India, be of 35 years of age or above. There is no written provision explaining that Indian Citizen of foreign origin as president.

Hence, Statement 2 is not correct.


90.Which one of the following factors constitutes the best safeguard of liberty in a liberal democracy?

  1. A committed judiciary
  2. Centralization of powers
  3. Elected government
  4. Separation of powers

Ans.(d). In the liberal approach which is inherent in democracy, all forms of power are rooted in the will of the people.

This enhances the rule of law as one of the basic foundations of democracy; it affirms the separation of powers as a vehicle for their restraint, and it promotes individuals’ rights and freedoms as a prerequisite for their dignity.

A “committed judiciary” is a derogatory word that means committed to the-then prevailing government. Hence (a) is wrong. Option (b) clearly is wrong.


91.Under the Indian Constitution, concentration of wealth violates

  1. the Right to Equality
  2. the Directive Principles of State Policy
  3. the Right to Freedom
  4. the Concept of Welfare

Ans.(b). In the directive principles of state policy of Indian Constitution, Article 38 says that “The State shall strive to promote the welfare of the people by securing and protecting a social order by ensuring social, economic and political justice and by minimising inequalities in income, status, facilities and opportunities”.

Articles 39: The State shall in particular, direct its policies towards securing:

Right to an adequate means of livelihood to all the citizens.

The ownership and control of material resources shall be organised in a manner to serve the common good.

The State shall avoid concentration of wealth in a few hands.

Equal pay for equal work for both men and women.

The protection of the strength and health of the workers.

Childhood and youth shall not be exploited.


92.What is the position of the Right to Property in India?

  1. Legal right available to citizens only
  2. Legal right available to any person
  3. Fundamental Right available to citizens only
  4. Neither Fundamental Right nor legal right

Ans.(b).

The Constitution originally provided for the ‘right to property’ under Articles 19 and 31 (fundamental right). Article 19 guaranteed to all citizens the right to acquire, hold and dispose of property. Article 31 provided that "no person shall be deprived of his property save by authority of law." It also provided that compensation would be paid to a person whose property has been taken for public purposes.

The provisions relating to the right to property were changed a number of times. The 44th Amendment of 1978 removed the right to property from the list of fundamental rights. A new provision, Article 300-A, was added to the constitution, which provided that "no person shall be deprived of his property save by authority of law".

Regarding foreigners and Indian citizens: A foreign national of non-Indian origin resident outside India cannot buy any immovable property in India. It is illegal for foreign nationals to own property in India unless they satisfy the residency requirement of 183 days in a financial year (a tourist visa lasts for 180 days). It is also illegal to buy property on a tourist visa.

Property cannot be purchased jointly in the name of one eligible person with one non-eligible person. That means a non-resident Indian (NRI) or foreign national of Indian origin (PIO) cannot buy a property jointly with a foreigner. However, a foreign national resident in India does not require approval of RBI to purchase any immovable property in India. This is because once he is a resident in India, he gets the rights like any other resident. This freedom is however not available to citizens of Pakistan, Bangladesh, Sri Lanka, Afghanistan, China, Iran, Nepal and Bhutan.


93.What was the exact constitutional status of India on 26th January, 1950?

  1. A Democratic Republic
  2. A Sovereign Democratic Republic
  3. A Sovereign Secular Democratic
  4. A Sovereign Socialist Secular Democratic Republic

Ans.(b). According to constitution at the time of its commencement on January 26, 1950, India was a Sovereign, Democratic, Republic. It got constitutional status of being Socialist and Secular after 42nd amendment act 1976.


94.Constitutional government means

  1. a representative government of a nation with federal structure
  2. a government whose Head enjoys nominal powers
  3. a government whose Head enjoys real powers
  4. a government limited by the terms of the Constitution

Ans.(d). Constitutional government is defined by the existence of a constitution—which may be a legal instrument or merely a set of fixed norms or principles generally accepted as the fundamental law of the polity—that effectively controls the exercise of political power.

Representative democracy, also known as indirect democracy, is a type of democracy founded on the principle of elected persons representing a group of people, in contrast to direct democracy.

A federal republic is a federation of states with a republican form of government.

Hence only option (d) is true.


95.With reference to India, the terms `Halbi, Ho and Kui' pertain to

  1. dance forms of Northwest India
  2. musical instruments
  3. pre-historic cave paintings
  4. tribal languages

Ans.(d). Halbi, Ho and Kui are the tribal languages of India.

Halbi is an Eastern Indo-Aryan language, transitional between Odia and Marathi. It is spoken by 500,000 people across the central part of India.

Ho is a Munda language of the Austroasiatic language family spoken primarily in India by about 1.04 million people (0.103% of India's population) per the 2001 census. It is spoken by the Ho, Munda, Kolha and Kol tribal communities of Odisha, Jharkhand, Bihar, Chhattisgarh, West Bengal, Assam and is written with the Warang Citi script.

Kui is a South-Eastern Dravidian language spoken by the Kandhas. It is mostly spoken in Odisha, and written in the Odia script.


96.Consider the following statements in respect of Bharat Ratna and Padma Awards :

  1. Bharat Ratna and Pad.-na Awards are titles under the Article 18(1) of the Constitution of India.
  2. Padma Awards, which were instituted in the year 1954, were suspended only once.
  3. The number of Bharat Ratna Awards is restricted to a maximum of five in a particular year.

Which of the above statements are not correct?

  1. 1 and 2 only
  2. 2 and 3 only
  3. 1 and 3 only
  4. 1,2 and 3

Ans.(c). Bharat Ratna -

  1. ‘Bharat Ratna’, the highest civilian Award of the country, was instituted in the year 1954.
  2. Any person without distinction of race, occupation, position or sex is eligible for these awards.
  3. It is awarded in recognition of exceptional service/performance of the highest order in any field of human endeavour.
  4. The recommendations for Bharat Ratna are made by the Prime Minister himself to the President. No formal recommendations for this are necessary.
  5. The number of annual awards is restricted to a maximum of three in a particular year.
  6. On conferment of the award, the recipient receives a Sanad (certificate) signed by the President and a medallion.
  7. The Award does not carry any monetary grant.

Padma Awards –

  1. The Padma Awards are one of the highest civilian honours of India announced annually on the eve of Republic Day.
  2. The Awards are given in three categories: Padma Vibhushan (for exceptional and distinguished service), Padma Bhushan (distinguished service of higher order) and Padma Shri (distinguished service).
  3. The award seeks to recognize achievements in all fields of activities or disciplines where an element of public service is involved.
  4. The Padma Awards are conferred on the recommendations made by the Padma Awards Committee, which is constituted by the Prime Minister every year. The nomination process is open to the public. Even self-nomination can be made.
  5. Padma Awards, which were instituted in the year 1954, is announced every year on the occasion of Republic Day except for brief interruption(s) during the years 1978 and 1979 and 1993 to 1997.

Article 18(1) of Indian Constitution abolishes all titles. It prohibits the State to confer titles on anybody whether a citizen or a non-citizen. Military and academic distinctions are, however, exempted from the prohibition. Thus, a university can give title or honor on a man of merit.

Hence statements 1, 2, and 3 are NOT correct.


97.Consider the following statements : Statement 1 :

Statement 1 :

The United Nations Capital Development Fund (UNCDF) and the Arbor Day Foundation have recently recognized Hyderabad as 2020 Tree City of the World.

Statement 2 :

Hyderabad was selected for the recognition for a year following its commitment to grow and maintain the urban forests.

Which one of the following is correct in respect of the above statements?

  1. Both Statement 1 and Statement 2 are correct and Statement 2 is the correct explanation for Statement 1
  2. Both Statement 1 and Statement 2 are correct but Statement 2 is not the correct explanation for Statement 1
  3. Statement 1 is correct but Statement 2 is not correct
  4. Statement 1 is not correct but Statement 2 is correct

Ans.(d). The Arbor Day Foundation and the UN’s Food and Agriculture Organisation (not UNCDF as given in question) have recognised Hyderabad as ‘2020 Tree City of the World’. So, statement 1 is not correct.

Hyderabad was selected as a Tree City because of its initiatives like Haritha Haram programme and Urban Forest Parks plan. Haritha Haram is a flagship program of the state government for large-scale plantations across the state to increase the green cover. Hence, statement 2 is correct.


98.Consider the following statements in respect of the Laureus World Sports Award which was instituted in the year 2000 :

  1. American golfer Tiger Woods was the first winner of this award.
  2. The award was received mostly by `Formula One' players so far.
  3. Roger Federer received this award maximum number of times compared to others.

Which of the above statements are correct?

  1. 1 and 2 only
  2. 2 and 3 only
  3. 1 and 3 only
  4. 1, 2 and 3

Ans.(c). The Laureus World Sports Awards is an annual award ceremony honouring individuals and teams from the world of sports along with sporting achievements throughout the year. It was established in 1999 by Laureus Sport for Good Foundation founding patrons Daimler and Richemont.

The first ceremony was held on 25 May 2000 in Monte Carlo in which American golfer Tiger Woods was the (first) winner of the award.

Swiss tennis player Roger Federer holds the record for the most awards with six, five for Sportsman of the Year and one for Comeback of the Year.

Of the total 21 times for which the award has been awarded yet the ‘Formula One’ players have won it only for four times i.e., in 2020 – Lewis Hamilton, 2014- Sebastian Vettel and 2002 and 2004 - Michael Schumacher.

Hence only statements 1 and 3 are true


99.Consider the following statements in respect of the 32nd Summer Olympics :

  1. The official motto for this Olympics is 'A New World'.
  2. Sport Climbing, Surfing, Skateboarding, Karate and Baseball are included in this Olympics.

Which of the above statements is/are correct?

  1. 1 only
  2. 2 only
  3. Both 1 and 2
  4. Neither 1 nor 2

Ans.(b). The official motto for the 32nd Summer Olympics (2020 Summer Olympics) held in Tokyo (Japan) in 2021 (delayed by the COVID-19 disease) was ‘United by Vision’.

There has been a total of 42 sports, spanning 55 disciplines, included in the Olympic programme at one point or another in the history of the Games. For 32nd Summer Olympics the schedule has comprised 33 sports, spanning 50 disciplines.

In 2016 Summer Olympics held at Rio de Janeiro (Brazil), there were 28 sports (41 disciplines). The disciplines added in 2020 Summer Olympics by the Japanese Olympic Committee were baseball and softball, karate, sport climbing, surfing and skateboarding, the last four of which made their Olympic debuts, and the last three of which will remain on the Olympic program.

Hence only statement 2 is true.


100.Consider the following statements in respect of the ICC World Test Championship

  1. The finalists were decided by the number of matches they won.
  2. New Zealand was ranked ahead of England because it won more matches than England.

Which of the above statements is/are correct?

  1. 1 only
  2. 2 only
  3. Both 1 and 2
  4. Neither 1 nor 2

Ans.(d). As per the information available on icc-cricket.com, New Zealand with 2,764 points has ranked ahead of England (points 3753).

For every test match contested in the 2021-23 cycle, in the ICC World Test Championship points system, twelve points will be awarded to the winning team, four points awarded for a draw and six for a tied game. The percentage of points won is used to determine the WTC standings.

Hence both 1 and 2 are not correct.



Exam Analysis HOME       2021 - Paper I - English      Paper II - E       Paper I - Hindi       Paper II - H      



Questions and Detailed Solutions are being continuously updated ... refresh and check. Comment and let us know your experience, answers and solutions too!